You are on page 1of 94

NAMA: …………………………………………………………..

KELAS: …………………………………………………………..

MODUL MATEMATIK TAMBAHAN (PANDUAN GURU)


TINGKATAN 5
PERSEDIAAN MENGHADAPI KERTAS 2
SPM 2022

BIL TOPIK MUKA SURAT CATATAN


6 SOALAN INDEX NUMBER 3–8
NOMBOR INDEKS

6 SOALAN LINEAR PROGRAMMING 9 – 19


PENGATURCARAAN LINEAR

5 SOALAN SOLUTION OF TRIANGLES 20 – 25


PENYELESAIAN SEGITIGA

5 SOALAN LINEAR LAW 26 – 35


HUKUM LINEAR

7 SOALAN PROBABILITY DISTRIBUTION 36 – 44


TABURAN KEBARANGKALIAN

10 SOALAN TRIGONOMETRIC FUNCTIONS 45 – 50


FUNGSI TRIGONOMETRI

6 SOALAN COORDINATE GEOMETRY 51 – 57


GEOMETRI KOORDINAT

5 SOALAN VECTOR 58 – 64
VEKTOR

7 SOALAN CIRCULAR MEASURES 65 - 72


SUKATAN MEMBULAT

6 SOALAN DIFFERENTIATION & INTEGRATION 73 – 80


PEMBEZAAN & PENGAMIRAN

6 SOALAN SYSTEMS OF EQUATIONS 81 – 83


SISTEM PERSAMAAN

7 SOALAN KINEMATICS OF LINEAR MOTION 84 – 91


KINEMATIK GERAKAN LURUS

5 SOALAN PROGRESSION 92 – 94
JANJANG

1
Rumus-rumus berikut boleh membantu anda menjawab soalan. Simbol-simbol yang diberi adalah yang biasa digunakan.
The following formulae may be helpful in answering the questions. The symbols given are the ones commonly used.

1
x=
16 sin (A  B) = sinA kos B  kos A sin B
sin (A  B) = sinA cos B  cos A sin B

2 loga b = 17 kos (A  B) = kos A kos B sin A sin B


cos (A  B) = cos A cos B sin A sin B
3 Tn = a + (n – 1)d
18 tan (A  B) =
4 Tn = ar n – 1

19 sin 2A = 2 sin A kos A


5 Sn = [2a + (n – 1)d]
sin 2A = 2 sin A cos A

6 Sn = = ,r1 20 kos 2A = kos2 A – sin2 A


= 2 kos2 A – 1
7 = 1 – 2 sin2 A
Z=

n r n–r
cos 2A = cos2 A – sin2 A
8 P(X = r) = Cr p q ,p+q=1 = 2 cos2 A – 1
= 1 – 2 sin2 A
9 n
Pr =
21 tan 2A =
10 n
Cr =
22 = =
11 I=  100
23 a2 = b2 + c2 – 2bc kos A
a2 = b2 + c2 – 2bc cos A
12 =
24 Luas segi tiga =
2 2
13 sin A + kos A = 1
Area of triangles =
sin2 A + cos2 A = 1

14 sek2 A = 1 + tan2 A
sec2 A = 1 + tan2 A

15 kosek2 A = 1 + kot2 A
cosec2 A = 1 + cot2A

2
ADDITIONAL MATHEMATICS SPM 2021
KERTAS 2 BAHAGIAN C: NOMBOR INDEKS
PAPER 2 SECTION C: INDEX NUMBER
Remember the formula…must memorise Ingat formula…wajib hafal
Price index, I =  100 Indeks harga, I =  100

Composite index, = Indeks gubahan, =

Cost of making:  100 = Composite index Kos pembuatan:  100 = Indeks gubahan

1 The diagram is a bar chart indicating the weekly cost of the items P, Q, R, S, and T for the year 1990. Table below
show the prices and the price indices for the item.
Rajah di bawah menunjukkan sebuah carta palang bagi kos mingguan untuk bahan P, Q, R, S, dan T untuk tahun
1990. Jadual di bawah menunjukkan harga bahan dan indeks harga bahan tersebut.
Item Price (Year 1990) Price (Year 1995) Price index
Bahan Harga (Tahun 1990) Harga (Tahun 1995) Indeks harga
P x RM0.70 Increase 75%
Q RM2.00 RM2.50
R RM4.00 RM5.50 y
S RM6.00 RM9.90 150
T RM2.50 z 120

(a) Find the value of x, y, and z / Cari nilai bagi x, y, and z.


(b) Calculate the composite index for the items in the year 1995 based on the year 1990.
Hitung indeks gubahan bagi bahan pada tahun 1995 berasaskan tahun 1990.
(c) The total monthly cost of the items in the year 1990 is RM456. Calculate the corresponding total monthly
cost for the year 1995 / Kos kos bulanan bagi bahan pada tahun 1990 ialah RM456. Kira kos bulanan yang
sepadan bagi tahun 1995.
(d) The cost of the items increase by 20% from the year 1995 to the year 2000. Find the composite index for the
year 2000 based on the year 1990.
Kos bahan tersebut meningkat sebanyak 20% dari tahun 1995 kepada tahun 2000. Cari indeks gubahan pada
tahun 2000 berasaskan tahun 1990.
Guided Solution / Bimbingan Menjawab
(a) For x / Untuk x For y / Untuk y For z / Untuk z
100 = Price index 100 = Price index 100 = Price
 100 = 100 + 75  100 = y index
 100 = 120
 100 = 175 y = 137.50
z = 3.00
x = 0.40 z = RM3.00
x = RM0.40
(b) Composite index for year 1995 based 1990, =
=
= 140.92
(c) Cost of making:  100 = Composite index
 100 = 140.92
Cost in year 1995 = RM 642.60

(d) New composite index = 140.92 


= 169.104
3
2 The table shows the price indices and percentage of usage of four items, P, Q, R, and S, which are the main
ingredients in the production of a type of biscuit.
Jadual menunjukkan indeks harga dan peratus bagi penggunaan bahan P, Q, R, dan S, yang mana merupakan
bahan utama bagi pengeluaran jenis biskut.
Item Price index for the year 1995 based on the year 1993 Percentage of usage (%)
Bahan Indeks harga untuk tahun 1995 berasaskan pada tahun 1993 Peratus penggunaan (%)
P 135 40
Q x 30
R 105
S 130 20
(a) Calculate / Hitung
(i) the price of S in the year 1993, if its price in the year 1995 is RM37.70. (Ans : 29)
harga bahan S pada tahun 1993, jika harga pada tahun 1995 ialah RM37.70.
(ii) the price index of P in the year 1995 based on the year 1991 if its price index in the year 1993 based on
the year 1991 is 120. (Ans : 162)
indeks harga bahan P pada tahun 1995 berasaskan tahun 1991 jika indeks harga pada tahun 1993
berasaskan tahun 1991 ialah 120.
(b) The composite index number of the cost of biscuit production for the year 1995 based on the year 1993 is
128, calculate
Indeks gubahan bagi pengeluaran biskut bagi tahun 1995 berasaskan tahun 1993 ialah 128, hitung
(i) the value of x (Ans: 125)
nilai bagi x
(ii) the price of a box of biscuit in the year 1993, if the corresponding price in the year 1995 is RM32.
harga bagi sekotak biskut pada tahun 1993, jika harga pada tahun 1995 ialah RM32. (Ans : 25)
Answer Space / Ruangan Jawapan
(a)(i) From  100 = Price index
 100 = 130
Price (year 1993) = RM29.00

(a)(ii) I1995/1991 = ? I1993/1991 = 120 From table: I1995/1993 = 135


I1995/1991 =  100  100 = 120  100 = 135
=   100
=   100
= 162

(b)(i) From composite index =


128 =
x = 125

(b)(ii) From  100 = Composite index


 100 = 128
Price in year 1993 = RM25

4
3 The table shows the price and price indices for the four ingredients, P, Q, R, and S, used in making biscuits of a
particular kind. The pie chart represents the relative amount of the ingredients P, Q, R, and S, used in making these
biscuits.
Jadual menunjukkan harga dan indeks harga bagi empat bahan, P, Q, R, dan S, digunakan dalam membuat
pelbagai jenis biskut. Carta pai berikut mewakilkan jumlah amaun bahan P, Q, R, dan S, dalam menghasilkan
biskut.
Item Price per kg Price index for the year 2004
Bahan Harga per kg based on the year 2001
Year 2001 Year 2004 Indeks harga pada tahun 2004
Tahun 2001 Tahun 2004 berasaskan tahun 2001
P 0.80 1.00 x
Q 2.00 y 140
R 0.40 0.60 150
S z 0.4 80
(a) Find the value of x, y and z, (Ans : x = 125, y = 2.80, z = 0.50)
Cari nilai bagi x, y dan z.
(b) (i) Calculate the composite index for the cost of making these biscuits in the year 2004 based on the year
2001. (Ans : 129.44)
Hitung indeks gubahan bagi kos pembuatan biskut pada tahun 2004 berasaskan tahun 2001.
(ii) Hence, calculate the corresponding cost of making these biscuits in the year 2001, if the cost in the year
2004 was RM2985. (Ans : 2306.09)
Oleh itu, kira kos yang sepadan bagi penghasilan biskut pada tahun 2001, jika kos pada tahun 2004
adalah RM2985.
(c) The cost of making these biscuits is expected to increase by 50% from the year 2004 to the year 2007. Find
the expected composite index for the year 2007 based on the year 2001. (Ans : 194.16)
Kos bagi penghasilan biskut dijangka meningkat sebanyak 50% dari tahun 2004 kepada tahun 2007. Cari
indeks gubahan pada tahun 2007 berasaskan tahun 2001.(Jwp : 194.16)
Answer Space / Ruangan Jawapan
(a) For x For y For z
 100 = x  100 = 140  100 = 80
x = 125 y = 2.80 z = 0.50

(b)(i) From composite index =


=
x = 129.44

(b)(ii) From  100 = Composite index


 100 = 129.44
Cost in year 2001 = RM2306.09

(c) Composite index in year 2004 = 129.44


Increase 50% to year 2007
Expected composite index in year 2007 = 129.44 
= 194.16
OR
Price in year 2004 = RM2985
Increase 50% to year 2007
Expected price in year 2007 = RM2985 
= RM4477.50
Expected composite index in year 2997 =  100
= 194.16
5
4 The table shows the prices and the price indices of five components, P, Q, R, S and T, used to produce a kind of toy.
The diagram shows a pie chart which represents the relative quantity of components used.
Jadual di bawah menunjukkan harga dan indeks harga bagi lima komponen, P, Q, R, S dan T, yang digunakan
dalam penghasilan sebuah mainan. Rajah menunjukkan carta pai yang mewakili kuantiti setiap bahan.
Component Price (RM) for the year Price index for the year 2006 based 2004
Komponen Harga (RM) untuk tahun Indeks harga bagi tahun 2006 berasaskan
2004 2006 2004
P 1.20 1.50 125
Q x 2.20 110
R 4.00 6.00 150
S 3.00 2.70 y
T 2.00 2.80 140
(a) Find the value of x and of y. (Ans : x = 2.00, y = 90)
Cari nilai bagi x dan of y.
(b) Calculate the composite index for the production cost of the toys in the year 2006 based on the year 2004.
Hitung indeks gubahan bagi kos penghasilan mainan pada tahun 2006 berasaskan tahun 2004. (Ans : 123.5)
(c) The price of each component increase by 20% from the year 2006 to the year 2008. Given that the production
cost of one toy in the year 2004 is RM55, calculate the corresponding cost in the year 2008. (Ans : 81.51)
Harga bagi setiap komponen meningkat sebanyak 20% dari tahun 2006 kepada tahun 2008. Diberi bahawa
kos penghasilan bagi sebuah mainan pada tahun 2004 ialah RM55, hitung harga yang sepadan pada tahun
2008.
Answer Space / Ruangan Jawapan
(a) For x For y
 100 = 110  100 = y
x = 2.00 y = 90

(b) From composite index =


=
x = 123.5

(c) Price of each component increase by 20%


New composite index = 123.5 
= 148.2

From  100 = Composite index


 100 = 148.2
 100 = 148.2
Cost in year 2008 = RM81.51

6
5 The table shows the prices and the price indices of four ingredients, fish, flour salt, and sugar, used to make a type
of fish cracker. The diagram shows a pie chart which represents the relative quantity of the ingredient used.
Jadual di bawah menunjukkan harga dan indeks harga bagi empat bahan, ikan, tepung, garam, dan gula, yang
digunakan bagi membuat keropok ikan. Rajah berikut menunjukkan sebuah carta pai yang mewakili kuantiti setiap
bahan yang digunakan.
Ingredients Price (RM) per kg for the year Price index for the year 2005 based 2004
Ramuan Harga (RM) per kg bagi tahun Indeks harga bagi tahun 2005
2004 2005 berasaskan 2004
Fish / Ikan 3.00 4.50 150
Flour / Tepung 1.50 1.80 h
Salt / Garam k 0.90 112.5
Sugar / Gula 1.40 1.47 105
(a) Find the value of h and of k. (Ans : h = 120, k = 0.80)
Cari nilai bagi h dan k.
(b) Calculate the composite index for the cost of making these crackers in the 2005 based on the year 2004.
Hitung indeks gubahan kos penghasilan keropok ikan pada tahun 2005 berasaskan tahun 2004. (Ans:126.375)
(c) The composite index for the cost of making these crackers increases by 50% from the year 2005 to the year
2009. Calculate
Indeks gubahan penghasilan keropok ikan meningkat sebanyak 50% dari tahun 2005 kepada tahun 2009.
Hitung
(i) the composite index for the cost of making these crackers in the year 2009 based on the year 2004,
indeks gubahan kos penghasilan keropok ikan pada tahun 2009 berasaskan tahun 2004, (Ans : 189.5625)
(ii) the price of a box of these crackers in the year 2009 if its corresponding price in the year 2004 is RM
25. (Ans : 47.39)
harga bagi sekotak keropok pada tahun 2009 jika harga yang sepadan pada tahun 2004 ialah RM 25.
Answer Space / Ruangan Jawapan
(a) For h For k
 100 = h  100 = 112.5
h = 120 k = 0.80

(b) From composite index =


=
x = 126.375

(c)(i) New composite index = 126.375 


= 189.5625

(c)(ii) From  100 = Composite index


 100 = 189.5625
 100 = 189.5625
Price in year 2009 = RM47.39

7
6 Rajah 6 menunjukkan indeks harga bagi empat jenis barangan runcit yang dijual di sebuah pasaraya pada tahun
2015 dan tahun 2020 berasaskan tahun 2010.
Diagram 6 shows the price indices of four types groceries of sold in a supermarket in the year 2015 and the year
2020 based on the year 2010.

Indeks harga
130 Price index

120

110

100
P Q R S Jenis barang
Rajah / Diagram 6 Items
(a) Hitung / Calculate
(i) harga bagi Q pada tahun 2015 jika harganya RM75 pada tahun 2010.
the cost of Q in the year 2015 if its cost RM 75 in the year 2010.
(ii) harga bagi R pada tahun 2010 jika harganya RM183 pada tahun 2020.
the cost of R in the year 2010 if it costs RM183 in the year 2020.
(b) Indeks gubahan bagi penggunaan empat barangan runcit tersebut ialah 108 pada tahun 2015 berasaskan tahun
2010, cari
The composite index of the usage of the four groceries is 108 in the year 2015 based on the year 2010 , find
(i) nilai bagi m jika nisbah bagi P : Q : R : S = 3 : m : m : 2m
the value of m if the ratio of P : Q : R : S = 3 : m : m : 2m
(ii) indeks gubahan pada tahun 2020 berasaskan tahun 2015.
the composite index in the year 2020 based on the year 2015. [(a)(i) RM78.75, (a)(ii) RM150, (b)(i) 2, (b)(ii) 102.86]
Answer Space / Ruangan Jawapan
(a)(i)  100 = 105
Price (year 2015) = 78.75
(a)(ii)  100 = 122
Price (year 2010) = RM150
(b)(i) From composite index =
108 =
108(3+ m + m + 2m) =
m=2

(b)(ii) From composite index (2020 based 2010) =

=
= 111.0909
Composite index (2020 based 2015) =   100

= 111.0909   100
= 102.8620

8
KERTAS 2 BAHAGIAN C: PENGATURCARAAN LINEAR
PAPER 2 SECTION C: LINEAR PROGRAMMING
Remember Ingat
Inequality Type of Ketaksamaan Jenis garisan
line
Is more than > ------------ Lebih daripada > ------------
Is less than < ------------ Kurang daripada < ------------
At least or is not less than  Sekurang-kurangnya atau tidak kurang daripada 
At most or is not more than ≤ Selebih-lebihnya atau tidak lebih daripada ≤
P exceeds Q by at most 8: P – Q ≤ 8 P melebihi Q selebih-lebihnya 8: P – Q ≤ 8

1 A district education office intends to organize a course on the teaching of Mathematics and Science in English. The
course will be attended by x Mathematics participants and y Science participants. The selection of participants is
based on the following constraints:
Suatu pejabat pendidikan daerah bercadang mengadakan kursus pengajaran Matematik dan Sains dalam bahasa
Inggeris. Kursus itu akan dihadiri oleh x peserta Matematik dan y peserta Sains. Pemilihan peserta adalah
berdasarkan kekangan berikut:
I. The total number of participants is at least 40.
Jumlah peserta sekurang-kurangnya 40 orang.
II. The number of Science participants is at most twice that of Mathematics.
Bilangan peserta Sains selebih-lebihnya dua kali bilangan peserta Matematik.
III. The maximum alloctation for the course is RM 7200. The expenditure for a Mathematics participants is RM 120
and for a Science participant is RM 80.
Peruntukan maksimum untuk perbelanjaan kursus tersebut ialah RM7200. Perbelanjaan untuk seorang peserta
Matematik ialah RM120 manakala perbelanjaan untuk seorang peserta Sains ialah RM80.
(a) Write down three inequalities, other than x  0 and y  0, which satisfy the above constraints.
Tuliskan tiga ketaksamaan selain, x  0 dan y  0, yang memenuhi semua kekangan di atas.
(b) Hence, by using a scale of 2 cm to 10 participants on both axes, construct and shade the region R which
satisfies all the above constraints.
Seterusnya, dengan menggunakan skala 2 cm kepada 10 orang peserta pada kedua-dua paksi, bina dan
lorekkan rantau R yang memenuhi semua kekangan di atas.
(c) Using the graph from (b), find
Dengan menggunakan graf anda dari (b), cari
(i) the maximum and minimum number of Mathematics participants when the number of Science
participants is 10. (Ans : xminimum = 31, xmaximum = 55)
bilangan maksimum dan minimum peserta Matematik apabila bilangan peserta Sains ialah 10 orang.
(ii) the minimum cost to run the course. (Ans : 3720)
kos minimum untuk mengadakan kursus tersebut.

Guided solution / Penyelesaian Terbimbing


(a) I: x + y  40 II: y  2x III: 3x + 2y  180
(b) For x + y  40 For y  2x For 3x + 2y  180
x 0 40 x 0 20 x 0 60
y 40 0 y 0 40 y 90 0
(c) (i) Minimum number of Mathematics participants: 32
Maximum number of Mathematics participants: 55
(ii) Minimum cost = 120x + 80y
At point (13, 27): Minimum cost = 120(13) + 80(27)
= RM3720
At point (26, 52): Minimum cost = 120(26) + 80(52)
= RM7280
So, the minimum cost to run the course is RM3720

9
10
2 Use graph paper to answer this question.
Guna kertas graf untuk menjawab soalan ini
An institution offers two computer courses, P and Q. The number of participants for course P is x and for course Q
is y. The enrolment of the participants is based on the following constraints:
Sebuah institusi menawarkan dua kursus komputer, P dan Q. Bilangan peserta bagi kursus P ialah x orang dan
bilangan peserta bagi kursus Q ialah y orang. Pengambilan peserta tidak melebihi 100 orang:
I. The total number of participants is not more than 100.
Jumlah peserta tidak melebihi 100 orang.
II. The number of participants for course Q is not more than 4 times the number of participants for course P.
Bilangan peserta kursus Q tidak melebihi 4 kali bilangan peserta kursus P.
III. The number of participants for course Q must exceed the number of participants for course P by at least 5.
Bilangan peserta kursus Q mesti melebihi bilangan peserta kursus P sekurang-kurang 5 orang.
(a) Write down three inequalities, other than x  0 and y  0, which satisfy all the above constraints.
Tuliskan tiga ketaksamaan selain, x  0 dan y  0, yang memenuhi semua kekangan di atas.
(b) Hence, by using a scale of 2 cm to 10 participants on both axes, construct and shade the region R which
satisfies all the above constraints.
Seterusnya, dengan menggunakan skala 2 cm kepada 10 orang peserta pada kedua-dua paksi, bina dan
lorekkan rantau R yang memenuhi semua kekangan di atas.
(c) Using the graph from (b), find
Menggunakan graf daripada (b), cari
(i) the range of the number of participants for course Q if the number of participants for course P is 30.
julat bilangan peserta bagi kursus Q jika bilangan peserta kursus P ialah 30 orang. (Ans : 35  y  70)
(ii) the maximum total fees per month that can be collected if the fees per month for course P and Q are
RM 50 and RM 60 respectively. (Ans : 5800)
jumlah yuran maksimum sebulan yang boleh dikutip jika yuran sebulan bagi kursus P dan kursus Q
masing-masing ialah RM50 dan RM60.
Answer Space / Ruangan Jawapan:
(a) I: x + y  100 II: y  4x III: y – x  5

(b) For x + y  100 For y  4x For y – x  5


x 0 50 x 0 20 x 0 20
y 100 50 y 0 80 y 5 25

(c)(i) 35  y  70

(c)(ii) Maximum total fee = 50x + 60y


At point (20, 80): Maximum fee = 50(20) + 60(80)
= RM5800 [Accepted]
At point (48, 53): Maximum fee = 50(48) + 60(53)
= RM5580 [Rejected]
So, the maximum fee that can be collected per month is RM5800

11
y

100

y = 2x
90

80 (20, 80)

70

60 y–x=5

(48, 53)
50 R

x + y = 100

40
Course Q

30

20

10

x
0 10 20 30 40 50 60 Course P

12
3 A workshop produces two types of rack, P and Q. The production of each type of rack involves two processes,
making and painting. The table shows the time taken to make and paint a rack of type P and a rack of type Q.
Sebuah bengkel perabot menghasilkan dua jenis rak, P dan Q. Penghasilkan setiap rak melibatkan dua proses, iaitu
membuat dan mengecat, jadual menunjukkan tempoh masa yang diambil untuk proses membuat dan mengecat
seunit rak P dan seunit rak Q.
Rack Time taken (minutes)
Rak Masa diambil (minit)
Making / Membuat Painting / Mengecat
P 60 30
Q 20 40
The workshop produces x racks of type P and y racks of type Q per day. The production of racks per day is based on
the following constraints:
Bengkel itu menghasilkan x unit rak P dan y unit rak Q sehari. Penghasilkan rak sehari adalah berdasarkan
kekangan berikut:
I: The maximum total time for making both racks is 720 minutes.
Jumlah masa maksimum untuk membuat kedua-dua rak adalah 720 minit.
60x + 20y  720 // 3x + y  36
II: The total time for painting both rack is at least 360 minutes.
Jumlah masa untuk mengecat kedua-dua rak adalah sekurang-kurangnya 360 minit.
30x + 40y  360 // 3x + 4y  36
III: The ratio of the number of racks of type P to the number of racks of type Q is at least 1 : 3.
Nisbah bilangan rak P kepada bilangan rak Q adalah sekurang-kurang 1 : 3.
x:y1:3

3x  y
y  3x [Accept 3x  y]
(a) Write down three inequalities, other than x  0 and y  0, which satisfy all the above constraints.
Tulis tiga ketaksamaan, selain x  0 dan y  0, yang memenuhi semua kekangan di atas.
(b) Using a scale of 2 cm to 2 racks on both axes, construct and shade the region R which satisfies all the above
constraints.
Menggunakan skala 2 cm kepada 2 unit rak pada kedua-dua paksi, bina dan lorek rantau R yang memenuhi
semua kekangan di atas.
(c) Using the graph from (b), find
Dengan menggunakan graf anda dari (b), cari
(i) the minimum number of racks of type Q if 7 racks of the type P are produced per day. (Ans : 4)
bilangan minimum rak Q jika bilangan rak P yang dihasilkan ialah 7 unit sehari.
(ii) the maximum total profit per day if the profit from one rack of type P is RM 24 and from one rack type
Q is RM 32. (Ans : 720)
jumlah keuntungan maksimum sehari jika keuntungan yang diperoleh dari satu unit rak P ialah RM24
dan dari satu unit rak Q ialah RM 32.
Answer Space / Ruangan Jawapan:
(b) For 3x + y  36 For 3x + 4y  36 For y  3x
x 6 12 x 0 12 x 0 4
y 18 0 y 9 0 y 0 12

(c)(i) Minimum number of racks of type Q = 3.75


= 4 racks

(c)(ii) Maximum profit per day = 24x + 32y


At point (2.2, 6.6): Maximum profit = 24(2.2) + 32(6.6)
= RM264 [Rejected]
At point (48, 53): Maximum profit = 24(6) + 32(18)
= RM720 [Accepted]
So, the maximum profit that can be collected per month is RM720

13
y

18 (6, 18)

16
Rack Q

y = 3x
14

3x + y = 36

12

10

R
8

(2.2, 6.6)

3x + 4y = 36

x
0 2 4 6 8 10 12 Rack P

14
4 A factory produces two components, P and Q. In a particular day, the factory produced x pieces of component P and
y pieces of component Q. The profit from the sales of a piece of component P is RM15 and a piece of component Q
is RM12. The production of the components per day is based on the following constraints:
Sebuah kilang menghasilkan dua komponen, P dan Q. Pada satu hari tertentu, kilang itu menghasilkan x keping
komponen P dan y keping komponen Q. Keuntungan daripada penjualan sekeping komponen P ialah RM15 dan
sekeping komponen Q ialah RM12. Penghasilan komponen-komponen itu dalam sehari adalah berdasarkan
kekangan berikut :
I: The total number of components produced is at most 500.
Jumlah komponen yang dihasilkan adalah selebih-lebihnya 500.
x + y  500
II: The number of component P produced is not more than three times the number of component Q.
Bilangan komponen P yang dihasilkan tidak melebihi tiga kali bilangan komponen Q.
x  3y
3y  x
III: The minimum total profit for both components is RM 4200.
Jumlah keuntungan minimum bagi kedua-dua komponen adalah RM 4200.
15x + 12y  4200
(a) Write three inequalities, other than x  0 and y  0, which satisfy all the above constraints.
Tulis tiga ketaksamaan, selain x  0 dan y  0, yang memenuhi semua kekangan di atas.
(b) Using a scale of 2 cm to 50 components on both axes, construct and shade the region R which satisfies all the
above constraints.
Menggunakan skala 2 cm kepada 50 komponen pada kedua-dua paksi, bina dan lorek rantau R yang
memenuhi semua kekangan di atas.
(c) Using the graph from (b), to find
Gunakan graf di (b) untuk mencari
(i) the minimum number of pieces of component Q if the number of pieces of component P produced on a
particular day is 100, (Ans : 225)
bilangan minimum komponen Q jika bilangan komponen P yang dihasilkan pada satu hari tertentu
adalah 100.
(ii) the maximum total profit per day. (Ans : 7125)
jumlah keuntungan maksimum dalam sehari.
Answer Space / Ruangan Jawapan:
(b) For x + y  500 For 3y  x For 15x + 12y  4200
x 0 250 x 0 300 x 0 280
y 500 250 y 0 100 y 350 0

(c)(i) Minimum number of pieces of component Q = 225

(c)(ii) Maximum profit per day = 15x + 12y


At point (220, 75): Maximum profit = 15(220) + 12(75)
= RM4200 [Rejected]
At point (375, 125): Maximum profit = 15(375) + 12(125)
= RM7125 [Accepted]
So, the maximum profit that can be collected per month is RM7125

15
y

500

x + y = 500

450
Component Q

400

350

300

250 R

200

150

(375, 125)
100

(220, 75)

50
15x + 12y = 4200
3y = x

x
0 150 200 300
50 100 250 Component P

16
5 Johan uses x small tiles and y big tiles for decoration of his bathroom.
The decoration budget is based on the following constraints:
Johan menggunakan x keping jubin kecil dan y keping jubin besar untuk hiasan bilik mandinya.
Perbelanjaan hiasan tersebut adalah berdasarkan kekangan berikut:
I: The total number of tiles connot exceed 100 pieces.
Jumlah bilangan jubin tidak boleh melebihi 100 keping.
x + y  100
II: The number of small tiles connot exceed three times the number of big tiles.
Bilangan jubin kecil tidak boleh melebihi tiga kali bilangan jubin besar.
x  3y
3y  x
III: The number of big tiles cannot exceed the number of small tiles by more than 30 pieces.
Bilangan jubin besar tidak boleh melebihi 30 bilangan jubin kecil.
y – x  30
(a) Write three inequalities, other than x  0 and y  0, which satisfy all the above constraints.
Tulis tiga ketaksamaan, selain x  0 dan y  0, yang memuaskan semua kekangan di atas.
(b) Using a scale of 2 cm to 10 tiles on both axes, construct and shade the region R which satisfies all the above
constraints.
Menggunakan skala 2 cm kepada 10 keping jubin pada kedua-dua paksi, bina dan lorek rantau R yang
memenuhi semua kekangan di atas.
(c) Using the graph from (b), to find
Menggunakan graf yang dibina di (b), cari
(i) the maximum number of small tiles that can be used, (Ans : 75)
bilangan maksimum jubin kecil yang boleh digunakan,
(ii) the maximum total cost for the tiles if the cost of a small tile is RM1.50 and the cost for a big tile is
RM3.00. (Ans : 247.5 135)
jumlah kos maksimum untuk jubin-jubin itu jika kos bagi sekeping jubin kecil itu ialah RM1.50 dan kos
bagi sekeping jubin besar itu ialah RM3.00.
Answer Space / Ruangan Jawapan:
(b) For x + y  100 For x  3y For y – x  30
x 0 50 x 0 30 x 0 10
y 50 0 y 0 10 y 30 40

(c)(i) Minimum number of small tiles = ---batal---

(c)(ii) Maximum profit per day = 1.50x + 3.00y


At point (10, 40): Maximum cost = 1.50(10) + 3.00(40)
= RM135 [Rejected]
At point (38, 13): Maximum cost = 1.50(38) + 3.00(13)
= RM96 [Accepted]
So, the maximum profit that can be collected per month is RM135

17
y

50
y – x = 30

40 (10, 40)

30
Big tiles

R
20
3y = x

(38, 13)
10

x + y = 50

0 10 20 30 40 50 x

Small tiles

18
6 Rajah 6 menunjukkan titik-titik yang ditanda pada Satah Cartes bagi mewakili rantau berlorek R.
Diagram 6 shows the dots marked on the Cartes plane to represent the shaded region R.

x=2 y=x+3

y=7

y=x–5

x=8
x+y=5

Rajah / Diagram 6
(a) Nyatakan lima enam ketaksamaan linear selain daripada x  0 dan y  0, yang mentakrifkan rantau R.
State five six linear inequalities besides x  0 and y  0, that define the shaded region R.
(b) Diberi titik (x, y) terletak dalam rantau R, cari nilai maksimum bagi 3x + 2y.
Given that the point (x, y) is situated in the region R, find the maximum value of 3x + 2y.
Answer Space / Ruangan Jawapan
(a) x + y  5 x2 x8 yx–5 y7 yx+3

(b) Try all the corners of the shaded region


Point (2, 3): Maximum value = 3(2) + 2(3)
= 12 [Rejected]
Point (2, 5): Maximum value = 3(2) + 2(5)
= 16 [Rejected]
Point (4, 7): Maximum value = 3(4) + 2(7)
= 26 [Rejected]
Point (8, 3): Maximum value = 3(8) + 2(3)
= 30 [Rejected]
Point (8, 7): Maximum value = 3(8) + 2(7)
= 38 [Accepted]

19
KERTAS 2 BAHAGIAN C: PENYELESAIAN SEGITIGA
PAPER 2 SECTION C: SOLUTION OF TRIANGLE
1 Ingat formula berikut:
Pithagoras theorem & Trigonometric formula Teorem Pithagoras & Formula Trigonometri

a2 + b2 = c 2 sin  = a2 + b2 = c2 sin  =
cos  = cos  =
tan  = tan  =
Sine rule Hukum sinus

Sine rule: the angle and length of side must be opposite Hukum sin: sudut dan panjang sisi mesti bertentangan
= = = =

Cosine rule Hukum kosinus

Cosine rule: the c and cos C must be opposite! Hukum kos: c dan cos C mesti bertentangan!
c2 = a2 + b2 – 2ab cos C c2 = a2 + b2 – 2ab cos C
Area of triangle Luas segitiga

Area = a b sin C Area = a b sin C Luas = a b sin C Luas = a b sin C

Heron’s formula Formula Heron

Count the value of s first Kira nilai s dahulu


s = (a + b + c) s = (a + b + c)

Area = Luas =

20
1 Segitiga PQRST pada Rajah 1 merupakan segitiga sama sisi dan setiap sisi berukuran 4 unit. Titik R ialah titik
tengah bagi garisan QS / The triangle PQRST in Diagram 1 is an equilateral triangle and each side measures 4 units. The
point R is the midpoint of the line QS.

Rajah / Diagram 1
(a) Hitung nilai kos bagi sudut yang berlorek pada gambar. Nyatakan jawapan anda dalam bentuk .
Calculate the cosine value of the shaded angle on the image. State your answer in the form of . [8 markah / marks]
(b) Garis lurus RT dipanjangkan ke titik T’ dengan keadaan PT = PT’. Pada rajah yang sama, lukis dan lorekkan
segitiga TPT’ / The straight line RT is extended to the point T ’with the condition PT = PT’. In the same figure, draw
and shaded TPT’s triangle. [2 markah / marks]
Guided Solution / Penyelesaian Terbimbing
(a) For equilateral triangle, RST = 60
1st: Calculate the length of RT: From cosine rule, RT2 = RS2 + TS2 – 2(RS)(TS) cos 60
RT2 = 22 + 32 – 2(2)(3) ( ) cos 60 = 0.5 =
=7
RT =
2nd: Find the TRS: From sine rule, =
=
Cross multiply: sin TRS = 3 sin 60
sin TRS = 3 ( )
sin TRS =
3rd: Find cos PRT: We know that PRT = 90 – TRS [cos(A B) = cos A cos B sin A sin B]
Taking cosine: cos PRT = cos [90 – TRS]
= cos 90 cos TRS + sin 90 sin TRS
= (0) cos TRS + (1) sin TRS
= sin TRS
=
=

= where a = 3, b = , c = 14
(b)

21
2 Diagram 2 shows a quadrilateral PQRS.
Rajah 2 menunjukkan satu sisiempat PQRS.

Diagram / Rajah 2
(a) Find / Cari
(i) the length, in cm, of QS,
panjang, dalam cm, bagi QS,
(ii) QRS,
(iii) the area, in cm2, of the quadrilateral PQRS. [8 marks / markah]
luas, dalam cm2, bagi sisiempat PQRS.
(b) (i) Sketch a triangle SQR which has a different shape from triangle SQR such that SR = SR, SQ = SQ
and SQR = SQR.
Lakarkan satu segitiga SQR yang mempunyai bentuk yang berlainan daripada segitiga SQR dengan
keadaan SR = SR, SQ = SQ dan SQR = SQR.
(ii) Hence, state SRQ. [2 marks / markah]
Answer Space / Ruangan Jawapan [(a)(i) 14.19, (a)(ii) 141.36, (a)(iii) 72.38, (b)(ii) 38.64]
(a)(i) P = 180 – 78 – 32
= 70
Use sine rule: =
=
QS = 14.1862 cm
(a)(ii) Use cosine rule: QS2 = SR2 + RQ2 – 2(SR)(RQ) cos R
14.18622 = 62 + 92 – 2(6)(9) cos R
2(6)(9) cos R = 62 + 92 – 14.18622
cos R =
cos R = – 0.7801
R = 141.28
So, QRS = 141.28
(a)(iii) Area of quadrilateral PQRS = Area of triangle PQS + Area of triangle SRQ
= (8)(14.1862)sin78 + (6)(9)sin141.28
= 72.3937 cm2
(b)(i)

(b)(ii) SRQ = 141.28


QR’R = 180 – 141.28 = 38.72
QRS’ = 38.72
22
3 Diagram 3 shows two triangles ABC and BDE.
Rajah 3 menunjukkan dua segitiga ABC dan BDE.

Diagram / Rajah 3
It is given that BE = 8.5 cm, DE = 4.6 cm and AC = 5.8 cm.
Diberi bahawa BE = 8.5 cm, DE = 4.6 cm dan AC = 5.8 cm.
(a) Calculate / Hitung
(i) the length, in cm, of BC, (Ans : 10.18)
panjang, dalam cm, bagi BC,
(ii) the length, in cm, of CD, (Ans : 2.2)
panjang, dalam cm, bagi CD,
(iii) the area, in cm2, of ABC. (Ans : 20.88) [8 marks / markah]
luas, dalam cm2, bagi ABC.
(b) (i) Sketch a ABC which has a different shape from ABC such that AB = AB, AC = AC and ABC
= ABC.
Lakarkan satu ABC yang mempunyai bentuk berlainan daripada ABC dengan keadaan AB = AB,
AC = AC dan ABC = ABC.
(ii) Hence, state the size of BAC. (Ans : 11) [2 marks / markah]
Seterusnya, nyatakan size bagi BAC.
Answer Space / Ruangan Jawapan [(a)(i) 10.18, (a)(ii) 2.2, (a)(iii) 20.88, (b)(ii) 11]
(a)(i) Guna =
 
=
BC = 10.1815 cm
(a)(ii) Form cosine rule: BD2 = BE2 + ED2 – 2(BE)(ED)cosBED
BD2 = 8.52 + 4.62 – 2(8.5)(4.6)cos140
BD2 = 153.3147
BD = 12.3820 cm
Then, length of CD = 12.3820 cm – 10.1815 cm
= 2.2005 cm
(a)(iii) Area of triangle ABC =  BC  AC  sin ACB
=  10.1815  5.8  sin (180 – 101 – 34)
= 20.8783 cm2

(b)(i)

(b)(ii) BAC = 180 – 34 – (180 – 45)


= 11

23
4 Rajah 4 di atas menunjukkan gambar sebuah bot, kapal terbang, stesen pelantar minyak dan juga seekor ikan jerung.
Diagram 4 above shows a picture of a boat, an airplane, an oil rig station and even a shark. [KBAT]

32
24 unit x

32 x

12 unit

Rajah / Diagram 4
Diberi jarak antara bot dan kapal terbang ialah 24 unit, serta jarak pelantar minyak dengan jerung ialah 12 unit
manakala sudut antara bot, kapal terbang dan pelantar minyak ialah 32º. Jarak antara pelantar minyak dan kapal
terbang adalah bersamaan dengan jarak bot dan pelantar minyak.
Given that the distance between the boat and the plane is 24 units, and the distance between the oil rig and the
shark is 12 units while the angle between the boat, the plane and the oil rig is 32º. The distance between the oil rig
and the aircraft is equal to the distance of the boat and the oil rig.
(a) Cari jarak antara kapal terbang dan pelantar minyak.
Find the distance between the plane and the oil rig.
(b) Seterusnya, hitung luas segitiga yang terhasil pada rajah di atas.
Hence, calculate the area of the resulting triangle in the figure above. [(a) 14.15 unit, (b) 169.38 173.0300unit2]
Answer Space / Ruangan Jawapan
(a) Let x = Distance between the plane and the oil rig
Angle between the ship-shark-plane = 180 – 32 – 32
= 116
From sine rule: =
x = 14.1501 cm

(b) Area of triangle =  24  (12 + 14.1501)  sin 32


= 173.0300 cm2

24
5 Rajah 5 menunjukkan Ali dan Karim menggunakan klinikometer untuk menentukan sudut dongakan tebing iaitu
sebanyak 37º. Ali berada di titik B, iaitu terletak sejauh 68.5 m dari Karim yang berada di titik D.
Diagram 6 shows Ali and Karim use a clinicometer to determine the elevation angle of the cliff which is 37º. Ali is
at point B, which is located 68.5 m from Karim who is at point D. [KBAT]

Rajah / Diagram 5
(a) Kira ketinggian tebing, h.
Calculate the height of the cliff, h.
(b) Ali dan Karim bercadang untuk melakukan aktiviti lasak ‘flying fox’ bersama rakan-rakannya. Mereka
bercadang untuk memasang tali dari titik A hingga titik D. Panjang tali yang dipunyai oleh mereka ialah 90m.
Adakah tali itu mencukupi? Berikan justifikasi anda.
Ali and Karim plan to do a ‘flying fox’ extreme activity with their friends. They plan to install the rope from
point A to point D. The length of the rope they own is 90m. Is the rope enough? Give your justification.
Answer Space / Ruangan Jawapan [(a) 36.03 m, (b) the rope is not enough because the length of AD exceed the length of the rope used]
(a) DCB = 180 – 41 – 62
= 77
Find the length of BC first: =
 
=
BC = 46.1222 cm
Find the value of h: tan 37 =
h = 34.7556 cm

(b) Find the length of AD: AD2 = AC2 + BC2


AD2 = 34.75562 + 46.12222
AD = 57.7513 cm
The rope is enough to do a flying fox.

25
KERTAS 2 BAHAGIAN B: HUKUM LINEAR
PAPER 2 SECTION B: LINEAR LAW
1 Jadual 1 menunjukkan nilai-nilai bagi dua pemboleh ubah x dan y, yang diperoleh daripada satu ujikaji. Pemboleh
ubah x dan y dihubungkan oleh persamaan y = h dengan keadaan h dan k ialah pemalar.
Table 1 shows the values for the two variables x and y, obtained from one experiment. The variables x and y are
related by equations y = h such that h and k are constants.
x 3.84 4.39 4.84 4.92 5.13
y 0.21 3.03 44.31 99.42 322.66
Jadual / Table 1
(a) Plot ln y melawan x – 32 dengan menggunakan skala 2 cm kepada 1 unit pada kedua-dua paksi. Seterusnya,
lukiskan garis lurus penyuaian terbaik.
Plot ln y against x – 32 by using the scale of 2 cm to 1 unit on both axes. Hence, draw the line of best fit.
(b) Gunakan graf anda di (a) untuk mencari nilai
Use your graph in (a) to find the values of
(i) h,
(ii) k,
(iii) y apabila x = 4.5.
y when x = 4.5
Answer Space / Ruangan Jawapan
(a) ln y – 1.56 1.11 3.79 4.60 5.78
(x – 3)2 0.71 1.93 3.39 3.69 4.54

(b) Ingat ln  loge


From y = h
1st Taking loge for both sides: loge y = loge h
2nd Separating loge: loge y = loge h + loge
3rd Jumping power: loge y = loge h + k(x2 – 6x + 9) loge e
= loge h + k(x – 3)2 (x2 – 6x + 9) = (x – 3)2 and loge e = 1
4th Rearrange loge y = k(x – 3)2 + loge h

5th Compare with Y = mX + c


(b)(i) loge h = c
loge h = – 2.5
h = – 0.00316

(b)(ii) k = m
k = 1.8182

(b)(iii) When x = 4.5


(x – 3)2 = 2.25
From graph, ln y = 1.7
y = 5.4739

26
ln y

(3.8, 4.5)

3
m=
= 1.8182

1.7

2.25
0 1 2 3 4 5 6 (x – 3)2

–1

(0.5, – 1.5)
–2

c = – 2.5
27
2 Pada tahun 1619, Johannes Kepler mendapati wujud perkaitan di antara purata jarak, dalam AU, planet-planet dari
Matahari dengan tempoh masa, dalam tahun, planet-planet tersebut mengelilingi Matahari pada orbit masing-masing
dalam satu kitaran. Hubungan di antara purata jarak, x dan tempoh, y diberikan oleh y2 = axn. Jadual 2 menunjukkan
hasil pencerapan yang dibuat oleh seorang pengkaji untuk menentusahkan teori yang diutarakan oleh Johannes
Kepler.
In 1619, Johannes Kepler found that there was a correlation between the average distance, in AU, of the planets
from the Sun and the period of time, in years, the planets orbited the Sun in their respective orbits in one cycle. The
relationship between the mean of the distance, x and the period, y is given by y2 = axn. Table 2 shows the results of
observations made by a researcher to verify the theory put forward by Johannes Kepler.
Planet Utarid Zuhrah Bumi Marikh Musytari Zuhal
Planet Mercury Venus Earth Mars Jupiter Saturn
Purata Jarak, x (dalam AU) 0.387 0.723 1.0 1.523 5.203 9.541
Mean Distance, x (in AU)
Tempoh, y (dalam Tahun) 0.241 0.615 1.0 1.881 11.861 29.457
Period, y (in Years)
Jadual 2 / Table 2
(a) Plot log10 y melawan log10 x dengan menggunakan skala 2 cm kepada 0.5 unit pada kedua-dua paksi.
Seteruskan lukiskan garis lurus penyuaian terbaik.
Plot log10 y against log10 x by using the scale of 2 cm to 0.5 units on both axes. Hence, draw the straight line
of best fit.
(b) Gunakan graf anda di 2(a) untuk mencari nilai a dan n.
Use your graph in 2(a) to find the value of a and of n.
(c) Sebuah planet baru ditemui dalam sistem suria dan didapati tempoh masa ia mengelilingi Matahari ialah
43.32 tahun. Tentukan purata jarak planet tersebut dalam integer terhampir, dalam km, dari Matahari jika 1
AU bersamaan 149 597 871 km.
A new planet has been discovered in the solar system and it is found that the period of time it orbits the Sun is
43.32 years. Determine the average distance to the closest integer, of the planet, in km, from the Sun if 1 AU
equals 149 597 871 km.
Answer Space / Ruangan Jawapan [(b) a = 1, n = 3, (c) 530788206 km]
(a) log10 y – 0.62 – 0.21 0.00 0.27 1.07 1.47
log10 x – 0.41 – 0.14 0.00 0.18 0.72 0.98

(b) From y2 = axn


1st: Taking log10 for both sides log10 y2 = log10 axn
2nd: Separating log10 log10 y2 = log10 a + log10 xn
rd
3 : Jump the power 2log10 y = log10 a + n log10 x
4th: Rearrange to Y = mX + c log10 y = log10 a + n log10 x
log10 y = n log10 x + log10 a

Y = mX + c

n=m and log10 a = c


n = 1.5 log10 a = 0
n=3 a=1

(c) Given y = 43.32 years


log10 y = 1.64
From graph, log10 x = 1.1
x = 12.5893 AU
= 12.5893  149597871 km
= 1883325614 km (Nearest integer)

28
y

2.0

1.5

m=
= 1.5
1.0

0.5

c=0

-0.5 0 0.5 1.0 1.5 x

-0.5

-1.0

29
3 Jadual 3 menunjukkan data yang diperoleh dari satu eksperimen yang melibatkan pemboleh ubah x dan pemboleh
ubah y. Kedua-dua pemboleh ubah ini dihubungkan oleh persamaan y = hx3 + kx2.
Table 3 shows the data obtained from an experiment involving the variable x and the variable y. These two
variables are related by equations y = hx3 + kx2.
x 0.5 1.0 1.5 2.0 2.5 3.0
y 0.31 2.05 6.19 14.00 26.30 45.00
Jadual / Table 3
(a) Terangkan salah satu cara suatu graf garis lurus boleh diperoleh dari persamaan tak linear yang
menghubungkan x dan y tersebut
Explain either one method in which a straight line graph can be obtained from the nonlinear equations
connecting x and y.
(b) Dengan menggunakan skala yang sesuai, plot graf bagi Y melawan X untuk kedua-dua kaedah yang
dinyatakan di (a). Seterusnya, lukis garis lurus penyuaian terbaik.
Using an appropriate scale, plot a graph of Y against X for both method described in (a). Hence, draw the
line of best fit.
(c) Cari nilai h dan k yang diperoleh daripada graf anda.
Find the value of h and k obtained from your graph?
Answer Space / Ruangan Jawapan
(a) From y = hx3 + kx2
Divide each term by x2: = hx + k
Plot against x will get a straight line

Divide each term by x3: =h+


= k( ) + h
Plot against will get a straight line

(b) 1.24 2.05 2.75 3.5 4.21 5.00


x 0.5 1.0 1.5 2.0 2.5 3.0

(c) h = gradient k = y-intercept


h = 1.51 k = 0.475

30
y
5.0

4.5

4.0

3.5

3.0

m=
= 1.51
2.5

2.0

1.5

1.0

0.5 c = 0.475

0 0.5 1.0 1.5 2.0 2.5 3.0 x

31
4 Guna kertas graf untuk menjawab soalan ini.
Use the graph paper to answer this question.
Jadual 4 menunjukkan dua pembolehubah, x dan y, yang diperolehi daripada suatu eksperimen. Pembolehubah-
pembolehubah x dan y dihubungkan oleh persamaan y = a2x2 + 2abx + b2, dengan keadaan a dan b adalah pemalar.
Table 4 shows the values of two variables, x and y, obtained from an experiment. The variables x and y are related
by the equation y = a2x2 + 2abx + b2, where a and b are constants.
x 2 4 6 8 10
y 40.23 25.81 14.58 6.54 1.68
Jadual / Table 4
(a) Ungkapkan y = a2x2 + 2abx + b2 ke dalam bentuk linear, Y = mX + c.
Express y = a2x2 + 2abx + b2 in linear form, Y = mX + c.
(b) Menggunakan skala 2 cm kepada 2 unit pada paksi-x dan 2 cm kepada 1 unit pada paksi- , plot
melawan x dan lukis satu garis lurus penyuaian terbaik.
Using a scale of 2 cm to 2 units on the x-axis and 2 cm to 1 unit on the -axis, plot against x and draw
the line of best fit.
(c) Menggunakan graf daripada 4(b),
Use the graph from 4(b),
(i) anggarkan nilai bagi a dan b.
estimate the value of a and b.
(ii) cari niai bagi x apabila y = 17.64.
find the value of x when y = 17.64.
Answer Space / Ruangan Jawapan [(c)(i) a = −0.6286, b = 7.6, (c)(ii) 5.4]
2 2 2
(a) y = a x + 2abx + b
= (ax + b)(ax + b)
y = (ax + b)2
Taking root mean square for both sides
= ax + b

Y = mX + c

(b) list the table first


6.34 5.08 3.82 2.56 1.30
x 2 4 6 8 10

(c)(i) a = gradient b = y-intercept


a = – 0.63 = 7.6

(c)(ii) When y = 17.64


= 4.2
From graph, x = 5.4

32
y

10

8
c = 7.6

m=

2 = – 0.63

0 2 4 6 8 10 x

33
5 Guna kertas graf untuk menjawab soalan ini.
Use the graph paper to answer this question.

Jadual 5 menunjukkan nilai-nilai daripada dua pembolehubah, x dan y yang diperolehi daripada suatu eksperimen.
Pembolehuba-pembolehubah x dan y dihubungkan oleh persamaan y = , dengan keadaan m dan n adalah
pemalar.
Table 5 shows the values of two variables, x and y, obtained from an experiment. The variables x and y are related
by the equation y = , where m and n are constants.
x 0.1 1.5 2.5 3.5 4.5 6.5
y 3.9 2.4 1.8 1.5 1.3 1.0
Jadual / Table 5
(a) Berdasarkan pada Jadual 5, bina satu jadual untuk nilai-nilai xy.
Based on the Table 5, construct a table for the values of xy.
(b) Plot xy melawan y, menggunakan skala 2 cm kepada 0.5 unit pada paksi mengufuk dan 2 cm kepada 1 unit
pada paksi mencancang. Seterusnya, lukis garis penyuaian terbaik.
Plot xy against y, using a scale of 2 cm to 0.5 units on the horizontal axis and 2 cm to 1 unit on the vertical
axis. Hence, draw the line of best fit.
(c) Menggunakan graf di 5(b) untuk mencari nilai bagi
Use the graph from 5(b) to find the value of
(i) m,
(ii) n,
(iii) kecerunan bagi garis lurus yang diperolehi jika dilukis melawan x.

the gradient of the straight line obtained if is plotted against x.


Answer Space / Ruangan Jawapan [(c)(i) 2.085, (c)(ii) 8.5, (c)(iii) 0.1176]
(a)
xy 0.39 3.60 4.50 5.25 5.85 6.50
y 3.9 2.4 1.8 1.5 1.3 1.0

(c) From y=
y(x + n) = m
xy + ny = m
xy = m – ny
xy = – ny + m

Compare: Y = mX + c

(c)(i) – n = gradient
– n = – 2.07
n = 2.07

(c)(ii) m = y-intercept
m = 8.5

(c)(iii) From y=
=
= x+
So, gradient =
=
= 0.1176

34
y

c = 8.5

m=
= – 2.07

x
0 0.5 1.0 1.5 2.0 2.5 3.0 3.5 4.0

35
KERTAS 2 BAHAGIAN B: TABURAN KEBARANGKALIAN
PAPER 2 SECTION B: PROBABILITY DISTRIBUTION
1 Remember / Ingat

Taburan Binomial / Binomial Distribution Taburan Normal / Normal Distribution


Saiz sampel = n Min = 
Sample size Mean = 

Kebarangkalian berjaya = p Sisihan piawai = 


Success probability = p Standard deviation = 
Kebarangkalian gagal = q Konsep min = nilai simetri dari bentuk loceng
Failure probability = q Concept of mean = symmetry value from bell-shape

Min,  = np
Mean,  = np

Varians, 2 = npq
Variance, 2 = npq
Min / mean,  = 35
Sisihan piawai,  =
Standard deviation,  = Skor-Z =
Z-score =
Ingat p + q = 1
Kebarangkalian bagi pembolehubah selanjar X yang
Kebarangkalian mendapat r daripada n kali cubaan berlaku antara a dan b ditulis sebagai P(a , X , b).
Probability of getting r from n trials Probability of a continuous random variable X that
P(X = r) = nCr pr q n – r occurs between a and b is written as P(a , X , b).
P(a < X < b) = P( <Z< )

36
THE UPPER TAIL PROBABILITY Q(z) FOR THE NORMAL DISTRIBUTION N(0,1)
KEBARANGKALIAN HUJUNG ATAS Q(z) BAGI TABURAN NORMAL N(0, 1)

37
1 (a) Kebarangkalian Aziz menang dalam suatu pertandingan memanah ialah 0.25.
Sebanyak n pertandingan dijalankan. Kebarangkalian dia menang tepat 4 pertandingan adalah tiga kali
kebarangkalian dia menang tepat 3 pertandingan.
The probability of Aziz wins in an archery competition is 0.25. The games are played for n times. The
probability he wins exactly 4 games are three times the probability he wins exactly 3 games.
(i) Cari nilai bagi n.
Find the value of n.
(ii) Hitung varians untuk bilangan Aziz kalah pertandingan.
Calculate the variance for the number of Aziz lose the game.
(b) Tinggi perempuan di sebuah bandar didapati bertaburan normal dengan min 165cm dan sisihan piawai 11.7cm.
In a town, the height of women are normally distribution with mean 165 cm and standard deviation 11.7 cm.
Cari / Find
(i) kebarangkalian bahawa tinggi seorang perempuan yang dipilih secara rawak kurang daripada 150 cm
the probability that the height of a woman selected at random is less than 150 cm.
(ii) tinggi seorang perempuan yang merupakan perempuan tertinggi di kalangan 10 orang.
the height of highest woman among the 10 women.
Answer Space / Ruangan Jawapan [(a)(i) 39, (a)(ii) 7.313, (b)(i) 0.09992, (b)(ii) 179.99]
(a)(i) p = 0.25 q = 0.75 dan guna formula P(X = r) = nCr pr q n – r
Kebarangkalian menang tepat 4 = 3  Kebarangkalian menang tepat 3
n
C4 (0.25)4 (0.75)n – 4 = 3  nC3 (0.25)3 (0.75) n –3

C4  nC3 =
n

 = 3  0.25 3 – 4  0.75 n – 3 – (n – 4)
 = 3  0.25 – 1 0.75 1
 =9
 
  
 =9
=9
n – 3 = 36
n = 39

(a)(ii) Varians = npq


= 39(0.25)(0.75)
= 7.3125

(b)(i) min,  = 165 sisihan piawai,  = 11.7 f (z)


P(X < 150) = P(Z < )
= P(Z < – 1.282)
= 0.09992 – 1.282 z

(b)(ii) P(X > x) = P(Z > ) 0.1


= P(Z > )
P(Z > ) = 0.1 z
= 1.282
x = 170.9994 cm
= 179.99 cm

38
2 (a) Satu pembolehubah X mempunyai taburan binomial dengan 10 percubaan di mana kebarangkalian berjaya
dalam setiap percubaan ialah p. Min bagi kejayaan itu ialah 4. Hitung
A random variable X has a binomial distribution with 10 trials where the probability of success in each trial is
p. The mean number of success is 4. Calculate
(i) nilai bagi p,
the value of p,
(ii) P(X ≤ 2)
(b) Diameter bagi limau daripada satu ladang mempunyai taburan normal dengan min 3.2 cm dan sisihan piawai
1.5 cm. Hitung
The diameter of limes from a farm has a normal distribution with a mean of 3.2 cm and a standard deviation
of 1.5 cm. Calculate
(i) kebarangkalian bahawa satu limau yang dipilih secara rawak daripada ladang ini mempunyai diameter
lebih daripada 3.9 cm,
the probability that a lime chosen at random from this farm has a diameter more than 3.9 cm,
(ii) nilai bagi d jika 33% daripada limau ini mempunyai diameter kurang daripada d cm.
the value of d if 33% of the limes have diameters less than d cm.
Answer Space / Ruangan Jawapan [(a)(i) 0.4, (a)(ii) 0.1673, (b)(i) 0.32025, (b)(ii) 2.54]
(a)(i) min = np
4 = 10p
p = 0.4

(a)(ii) P(X ≤ 2) = P(X = 0) + P(X = 1) + P(X = 2)


= 10C0 (0.4)0 (0.6)10 + 10C1 (0.4)1 (0.6)9 + 10C2 (0.4)2 (0.6)8
= 0.1673

(b)(i) min,  = 3.2 sisihan piawai,  = 1.5 f (z)


P(X > 3.9) = P(Z > )
= P(Z > 0.4667)
= 0.32025 0.4667 z

(b)(ii) min,  = 3.2 sisihan piawai,  = 1.5 f (z)


P(X < d) = 33% 0.33
P(Z < ) = 0.33
= – 0.44 z
d = 2.54

39
3 (a) Didapati bahawa 70% daripada murid-murid daripada suatu kelas memperolehi gred A dalam Geografi dalam
peperiksaan percubaan SPM. Jika 10 orang murid daripada kelas itu dipilih secara rawak, cari kebarangkalian
bahawa
It is found that 70% of the students from a certain class obtained grade A in Geography in SPM trial
examination. If 10 students from the class are selected at random, find the probability that
(i) tepat 8 orang murid memperolehi gred A,
exactly 8 students obtained grade A,
(ii) tidak lebih daripada 8 orang murid memperolehi gred A.
not more than 8 students obtained grade A.
(b) Cumulative Grade Point Average (CGPA) bagi tahun terakhir pelajar-pelajar di sebuah universiti mengikuti
satu taburan normal dengan min 2.6 dan sisihan piawai 0.25.
The Cumulative Grade Point Average (CGPA) of the final year students in a university follows a normal
distribution with a mean of 2.6 and a standard deviation of 0.25.
(i) Jika seorang pelajar dipilih secara rawak, cari kebarangkalian bahawa CGPA bagi pelajar itu adalah
lebih daripada 3.0.
If one student is randomly selected, find the probability that the CGPA of the student is more than 3.0.
(ii) Satu ijazah akan diberi kepada pelajar tahun akhir yang memperolehi CGPA lebih daripada k. Jika
89.5% daripada pelajar-pelajar berjaya memperolehi ijazah itu, cari nilai bagi k.
A degree will be given to the final year students who obtained CGPA more than k. If 89.5% of the
students manage to get a degree, find the value of k.
Answer Space / Ruangan Jawapan [(a)(i) 0.2335, (a)(ii) 0.8507, (b)(i) 0.0548, (b)(ii) 2.287]
(a)(i) P(X = 8) = 10C8 (0.7)8 (0.3)2
= 0.2335

(a)(ii) P(X ≤ 8) = 1 – P(X = 9) + P(X = 10)


= 1 – 10C9 (0.7)9 (0.3)1 – 10C10 (0.7)10 (0.3)0
= 0.8507

(b)(i) min,  = 2.6 sisihan piawai,  = 0.25 f (z)


P(X > 3.0) = P(Z > )
= P(Z > 1.6)
= 0.0548 1.6 z

(b)(ii) min,  = 2.6 sisihan piawai,  = 0.25 f (z)


P(X > k) = 89.5% 0.105 0.895
P(Z > ) = 0.895
= – 1.253 z
k = 2.28675

40
4 Satu kajian menunjukkan bahawa baki kad kredit bagi pelanggan-pelanggan adalah bertaburan normal seperti
ditunjukkan dalam Rajah 4.
A study shows that the credit card balance of the customers is normally distributed as shown in Diagram 4.

Rajah / Diagram 4
(a) (i) Cari sisihan piawai.
Find the standard deviation.
(ii) Jika 30 orang pelanggan dipilih secara rawak, cari bilangan pelanggan yang mempunyai baki kad kredit
antara RM1800 dan RM3000.
If 30 customers are chosen at random, find the number of customers who have the credit card balance
between RM1800 and RM3000.
(b) Didapati bahawa 25% daripada pelanggan mempunyai baki kad kredit kurang daripada RMy. Cari nilai bagi y.
It is found that 25% of the customers have the credit card balance less than RMy. Find the value of y.
Answer Space / Ruangan Jawapan [(a)(i) 900, (a)(ii) 13, (b) RM2263.40]
(a)(i) mean,  = 2870
P(X > 3770) = 15.87%
P(Z > ) = 0.1587
From table, Q(0.1587) = 1.00
So, = 1.00
 = 900

(a)(ii) P(1800 < X < 3000) = P( <Z< )


= P(2.1888 < Z < 0.1444)
= 0.4427 – 0.0143
= 0.4284
Then, = 0.4284
Number of customer with that credit balance = 12.852
= 13 customers

(b)(i) P(X < y) = 25%


P(Z < ) = 0.25
From table, Q(0.25) = 0.674
So, = – 0.674
y = 2263.4
= RM2263.40

41
5 Kelab Memanah telah menganjurkan satu Hari Terbuka Memanah bagi mengutip dana untuk kelab mereka. Papan
sasaran mereka adalah seperti Rajah 5.
Archery Club is organizing The Archery Open Day to raise a fund for their club. The target board is shown in
Diagram 5.

Rajah / Diagram 5
Kelab tersebut menawarkan RM 5 untuk 5 anak panah bagi sekali permainan. Hamper alat tulis akan diberi kepada
mereka yang berjaya memanah kesemua 5 anak panah tepat ke kawasan berwarna kuning di papan sasaran. Adam
seorang pemanah ingin mencuba mendapatkan hamper tersebut. Rekod Adam secara puratanya ialah dia boleh
membuat 8 panahan tepat ke kawasan kuning dari 10 panahan yang dibuat.
The club offer RM5 for 5 arrows per game. A stationery hamper will be given for those who can shoot all 5 arrows
right to the yellow area at the target board. Adam is an archers. He would like to try to win the hamper. Based on
Adam’s record, by average Adam can shoot 8 arrows right to the yellow area out of 10 arrows.
(a) Adam mempunyai RM40 dan dia akan mencuba bermain hanya sekiranya dia mempunyai sekurang-kurangnya
95% peluang untuk memenangi hamper tersebut.
Tunjukkan pengiraan matematik bagi membantu Adam membuat keputusan samada untuk mencuba bermain
atau tidak.
Adam has RM40 and he will only try the game if he had at least 95% chance to win the hamper. Show the
mathematical calculation to help Adam make his decision whether to play or not. [7 markah / marks]
(b) Berapakah jangkaan bilangan minimum permainan yang Adam perlukan sekiranya dia ingin memenangi 3
hamper tersebut?
What is the estimated minimum number of games that Adam needed if he wants to get 3 hampers? [3 marks]
Answer Space / Ruangan Jawapan [(a) ya, (b) 9]
(a) Adam can shoot 8 out of 10  p = 0.8 q = 0.2
Calculating the probability to hit all the 5 arrows to the yellow area at the target
P(hit all 5 arrows) = P(1st hit and 2nd hit and 3rd hit and 4th hit and 5th hit)
= 0.8  0.8  0.8  0.8  0.8
= 0.32768
So, p = all 5 arrows must hit the target q = contain mistake of hitting the target
= 0.32768 = 1 – 0.32768
= 0.67232
Calculating the probability that Adam can win the hamper [At least winning one hamper]
He has RM40 meaning that he can try 8 games.
P(He get at least one hamper) = 1 – P(not winning at all)
= 1 – 8C0 (0.32768)0 (0.67232)8
= 0.9583
= 95.83% (He can proceed to play)

(b) P(hit all 5 arrows to target) = 0.32768


Then, = 0.32768
= 0.32768
n = 9.1553
Adam must try at least 9 times in order to get three hampers.

42
6 Masa untuk sebuah van yang menghantar pekerja kilang sampai ke kilangnya ditunjukkan di dalam graf taburan
normal dalam Rajah 6.
The graph shows the normal distribution of the time for a van that brings the industrial workers to their factory as
shown in Diagram 6.

Rajah / Diagram 6
(a) (i) Nyatakan nilai min, . [1 markah / mark]
State the value of mean, .
(ii) Hitung nilai sisihan piawai bagi taburan normal tersebut. [3 markah / marks]
Calculate the value of standard deviation for that normal distribution.
(b) Min masa untuk van tersebut sampai ke kilang ialah 7.30 pagi bagi shif siang. Van akan dianggap lewat
sekiranya ia sampai selepas 7.40 pagi dan pihak pengurusan van akan dikenakan bayaran penalti.
The mean time for the van to arrive at factory is 7.30 am for the day shift. an will be considered late if its
arrived after 7.40 am and the management of the van will be charged penalty.
(i) Hitung kebarangkalian pihak pengurusan van akan dikenakan penalti. [3 markah / marks]
Calculate the probability the management of the van will be charged penalty.
(ii) Jika 60% perkhidmatan van tersebut sampai k minit sebelum atau selepas 7.30 pagi, cari julat waktu van
tersebut tiba di kilang. Berikan julat waktu tepat kepada saat yang terhampir.
If 60% of the van's service arrives k minutes before or after 7.30 am, find the time range for the van to
arrive at the factory. Give the exact time range to the nearest second. [3 markah / marks]
Answer Space / Ruangan Jawapan [(a) 3.86 minutes, (b)(i) 0.00478, (b)(ii) 7:26:45am – 7:33:15am]
(a)(i) Mean,  = 0

(a)(ii) Standard deviation = 


From the graph, P(X > 4) = 50% – 35%
P(X > 4) = 15%
P(Z > ) = 0.15
From table, Q(0.15) = 1.036, then = 1.036
 = 3.8610 minutes

(b)(i) Mean,  = 7.30, Standard deviation =  = 3.8610


P(X > 7.40) = P(Z > )
= P(Z > 2.590)
= 0.00480

(b)(ii) P(k < X < k ) = 0.60


P(X > k) = 0.20
P(Z > ) = 0.20
From table, Q(0.20) = 0.842
So, = 0.842
k = 3.25 minutes
= 3 minutes 15 seconds
Time range = 7:33:15am atau 7:26:45am

43
7 (a) Dalam industri kelapa sawit, buah tersebut akan dikategorikan mengikut bilangan soket segar buah relai dalam
satu tandannya. Didapati, bilangan soket segar tersebut adalah bertaburan secara normal dengan min 10 soket
dan sisihan piawai 4 soket. Buah kelapa sawit tersebut dikategorikan kepada 3 kategori mengikut bilangan
soket segar buah relainya seperti Jadual 7.
In palm fruit industry, the fruit will be categorize according to the numbers of fresh loose socket in one bunch of the palm fruit. It was
found that, numbers of fresh loose socket are normally distributed with a mean of 10 sockets and a standard deviation 4 sockets. There
are 3 category of the palm fruit base on their numbers of fresh loose socket as shown in Table 7.
Kategori Jumlah soket segar buah relai
Category Numbers of fresh loose socket
Tandan Masak/Ripe Bunch Lebih banyak/More
Tandan Mengkal/Underripe Bunch Sederhana/Average
Tandan Muda/Unripe Bunch Sedikit/Less
Jadual / Table 7
(i) Bilangan soket segar buah relai dianggap lebih banyak sekiranya bilangannya adalah sekurang-
kurangnya sebanyak 13 soket. Jika satu tandan buah kelapa sawit dipilih secara rawak, cari
kebarangkalian bahawa buah kelapa sawit itu adalah kategori Tandan Masak.
Numbers of fresh loose socket is considered as more if its quantity is at least 13 sockets. If a bunch of palm fruit is picked at
random, find the probability that the palm fruit is in the Ripe Bunch category.
(ii) Cari bilangan soket segar buah relai minimum untuk kategori Tandan Mengkal jika 10% daripada buah
kelapa sawit itu adalah di dalam kategori Tandan Muda.
Find the minimum numbers of fresh loose socket for Underripe Bunch category if 15% 10% of the palm fruit are in the Unripe
Bunch.
(b) Alia menghadiri satu kelas memasak pada setiap hari Ahad. Kebarangkalian dia dapat menghadirkan diri ke
kelas tersebut ialah 0.85. Kelas tersebut diadakan 4 kali sebulan. Sijil penyertaan akan diberikan kepada
pelajar yang dapat menyertai kelas sekurang-kurangnya 3 kali sebulan.
Alia attends a cooking class every Sunday. The probability she is able to attend the class is 0.85. The class is held 4 times a month.
Student will be given a certificate of participation if they can attend the class at least 3 times a month.
(i) Hitung kebarangkalian Alia akan mendapat sijil penyertaan pada sesuatu bulan.
Calculate the probability that Alia will get the certificate of participation on a particular month .
(ii) Kelas tersebut diadakan selama 8 bulan. Alia akan berpeluang menduduki ujian akhir bagi
melayakkannya mendapat sijil pengiktirafan sekiranya dia mempunyai sekurang-kurangnya 6 sijil
kehadiran. Hitung kebarangkalian Alia boleh mengambil ujian akhir tersebut.
The class lasts for 8 months. Alia is allowed to sit for the final test to get the certificate of acknowledgement if she had at least 6
certificates of attendance. Calculate the probability that Alia qualifies to sit for the final test .
Answer Space / Ruangan Jawapan [(a)(i) 0.2266, (a)(ii) 5, (b)(i) 0.89, (b)(ii) 0.951]
(a)(i) P(X > 13) = P(Z > )
= P(Z > 0.75)
= 0.2266
(a)(ii) P(X < x) = 10%
P(Z < ) = 0.10
From table, Q(0.10) = 1.282
So, = – 1.282
x = 4.872
Minimum number = 5 sockets

(b)(i) n = 4, p = 0.85, q = 0.15


P(at least 3) = P(X = 3) + P(X = 4)
= 4C3 (0.85)3 (0.15)1 + 4C4 (0.85)4 (0.15)0
= 0.8905
(b)(ii) Pobability that Alia gets the attendance certificate in one month, p = 0.8905
Probability that Alia fails to get the attendance certificate in one month, p = 0.1095
She qualifies for the exam if she can get 6 out of 8 certificate.
P(X  6) = P(X = 6) + P(X = 7) + P(X = 8)
= 8C6 (0.8905)6 (0.1095)2 + 8C7 (0.8905)7 (0.1095)1 + 8C8 (0.8905)8 (0.1095)0
= 0.9518

44
KERTAS 2 BAHAGIAN B/A: FUNGSI TRIGONOMETRI
PAPER 2 SECTION B/A: TRIGONOMETRIC FUNCTION
1 Ingat bentuk asas bagi graf trigonometri dalam 2
Remember the general shape for trigonometric graph in 2
Bentuk asas graf bagi y = sin x Bentuk asas graf bagi y = cos x Bentuk asas graf bagi y = tan x
Basic graph for y = sin x Basic graph for y = cos x Basic graph for y = tan x

Satu gelombang dalam 2 Bentuk sayap in 2 4 kawasan: +-+-


Wing-shape in 2 4 regions: +-+-
One wave in 2
Contoh / Example 1
Lakarkan graf bagi y =  tan 2x  for 0  x  2π
Sketch the graf for y =  tan 2x  for 0  x  2π

 Amplitude = 1 Two general shapes in 2π Negative graph is reflected about the x-axis


y = tan 2x y =  tan 2x 
Contoh / Example 2
Lakarkan graf bagi y = cos 2x + 1
Sketch the graph for y = cos 2x + 1

Two general shapes Shift the x-axis 1 unit down

Contoh / Example 3
Lakarkan graf bagi y = sin x – 1  Amplitude: sin: waves x-axis shifted up by 1 unit
Sketch the graph for y = sin x – 1

y = sin x y = sin x – 1
45
1 (a) Lakarkan graf y = −2 cos x untuk 0  x  2.
Sketch the graph y = −2 cos x for 0  x  2.
(b) Seterusnya, dengan menggunakan paksi-paksi yang sama, lakarkan satu graf yang sesuai untuk mencari
bilangan penyelesaian kepada persamaan + 2 cos x = 0 for 0  x  2. Nyatakan bilangan penyelesaian itu.
Hence, using the same axes, sketch a suitable graph to find the number of solutions to the equation
+ 2 cos x = 0 for 0  x  2. State the number of solutions.
Answer Space / Ruangan Jawapan [(b) y = , 2]
(a) From y = −2 cos x
Graph with amplitude: – 2 , shape U and cycle is 1U in 2
y

0  2 x

–2 y = – 2cos x

(b) From + 2 cos x = 0 -------(1)


From y = −2 cos x --------(2)
Substitute (2) into (1): − y = 0
y=

x  2
y 1 0.5
So, there are 2 solutions.

2 (a) Lakarkan graf bagi y =  3 cos 2x  untuk 0  x  2.


Sketch the graph of y =  3 cos 2x  for 0  x  2.
(b) Seterusnya, dengan menggunakan paksi-paksi yang sama, lakarkan satu graf yang sesuai untuk mencari
bilangan penyelesaian kepada persamaan 2 −  3 cos 2x  = for 0  x  2.
Nyatakan bilangan penyelesaian itu.
Hence, using the same axes, sketch a suitable straight line to find the number of solutions for the equation
2 −  3 cos 2x  = for 0  x  2. State the number of solutions.
Answer Space / Ruangan Jawapan [(b) y = 2 - , 8]
(a) Amplitude = 3, shape U and cycle is 2U in 2. Final sketch is reflected up about the x-axis
y

3 y =3cos 2x

0  2 x

–3
(b) From 2 −  3 cos 2x  = ----------(1)
From y =  3 cos 2x  ---------(2)
Substitute (2) into (1): 2 – y =
y=2–
x 0 2
y 2 0
There are 8 solutions.
46
3 (a) Lakarkan graf bagi y = cos 2x untuk 0  x   / Sketch the graph of y = cos 2x for 0  x  .
(b) Seterusnya, dengan menggunakan paksi-paksi yang sama, lakarkan satu graf yang sesuai mencari bilangan
penyelesaian kepada persamaan x – cos 2x = for 0  x  . Nyatakan bilangan penyelesaian itu.
Hence, using the same axes, sketch a suitable straight line to find the number of solutions for the equation
x – cos 2x = for 0  x  . State the number of solutions.
Answer Space / Ruangan Jawapan [(b) y = , 3]

(a) Amplitude = Shape = U and cycle 2U in 2


y

1.5 y =1.5cos 2x

0  1.5 2 x

– 1.5

(b) From x – cos 2x = -----------(1)


From y = cos 2x
y = cos 2x -----------(2)
Substitute (2) into (1): x– y=
y= x–
y= x–
x 0 1.5
y – 2.25 0.75
There are 3 solutions.
4 (a) Lakarkan graf bagi y = – 3 sin x untuk 0  x  2 / Sketch the graph of y = – 3 sin x for 0  x  2.
(b) Seterusnya, dengan menggunakan paksi-paksi yang sama, lakarkan satu graf yang sesuai untuk mencari
bilangan penyelesaian kepada persamaan + 3 sin x = 0 for 0  x  2. Nyatakan bilangan penyelesaian itu.
Hence, using the same axes, sketch a suitable straight line to find the number of solutions for the equation
+ 3 sin x = 0 for 0  x  2. State the number of solutions.
Answer Space / Ruangan Jawapan [(b) y = , 2]
(a) Amplitude = 3 Shape = wave and cycle 1.5waves in 2. Final sketch is reverse.
y

0  1.5 2 x

–3

(b) From + 3 sin x = 0 -----------(1) From y = – 3 sin x ----------(2)


Substitute (2) into (1): –y=0
y=
x  1.5
y 1 0.67
There are 2 solutions
47
5 (a) Lakarkan graf bagi y = 3 sin 2x untuk 0  x  2.
Sketch the graph of y = 3 sin 2x for 0  x  2.
(b) Seterusnya, dengan menggunakan paksi-paksi yang sama, lakarkan satu graf yang sesuai untuk mencari
bilangan penyelesaian kepada persamaan x =  (1 – sin 2x) for 0  x  2.
Nyatakan bilangan penyelesaian itu.
Hence, using the same axes, sketch a suitable straight line to find the number of solutions for the equation
x =  (1 – sin 2x) for 0  x  2. State the number of solutions.
Answer Space / Ruangan Jawapan [(b) y = – x + 3 , 5 ]
(a) Amplitude = 3 Shape = wave and cycle 2waves in 2.
y

0  2 x

–3

(b) From x =  (1 – sin 2x) From y = 3 sin 2x ----------(2)


x =  –  sin 2x -----------(1)
Substitute (2) into (1): x= – y
y= –x
y=3–
x 0 2
y 3 –1
There are 5 solutions

6 (a) Lakarkan graf bagi y =  – sin 2x untuk 0  x  2.


Sketch the graph of y =  – sin 2x for 0  x  2.
(b) Seterusnya, dengan menggunakan paksi-paksi yang sama, lakarkan satu graf yang sesuai untuk mencari
bilangan penyelesaian kepada persamaan 2  sin 2x  – 1 = 0 for 0  x  2.
Nyatakan bilangan penyelesaian itu.
Hence, using the same axes, sketch a suitable straight line to find the number of solutions for the equation
2  sin 2x  – 1 = 0 for 0  x  2. State the number of solutions.
Answer Space / Ruangan Jawapan [(b) y = ½ , 8]
(a) Amplitude = 1 Shape = wave and cycle 2waves in 2. Final graph is all positive
y

0  2 x

–1

(b) From 2  sin 2x  – 1 = 0 ---------(1) From y =  – sin 2x ----------(2)


Substitute (2) into (1): 2y – 1 = 0
y=
There are 8 solutions

48
7 (a) Lakarkan graf bagi y = 1 – sin 2x untuk 0  x  2.
Sketch the graph of y = 1 – sin 2x for 0  x  2.
(b) Seterusnya, dengan menggunakan paksi-paksi yang sama, lakarkan satu graf yang sesuai untuk mencari
bilangan penyelesaian kepada persamaan x = 2 sin 2x for 0  x  2.
Nyatakan bilangan penyelesaian itu.
Hence, using the same axes, sketch a suitable straight line to find the number of solutions for the equation
x = 2 sin 2x for 0  x  2. State the number of solutions.
Answer Space / Ruangan Jawapan [(b) y = – x + 1 , 4]
(a) Amplitude = 1 Shape = wave and cycle 2waves in 2. [Reverse the graph, then shift x-axis down by 1 unit]
y

0  2 x

(b) From x = 2 sin 2x ---------(1) From y = 1 – sin 2x


sin 2x = 1 – y -----------(2)
Substitute (2) into (1): x = 2 (1 – y)
(1 – y) =
y=1–
x 0 2
y 1 0
There are 4 solutions

8 (a) Lakarkan graf bagi y = 3 tan x untuk 0  x  2.


Sketch the graph of y = 3 tan x for 0  x  2.
(b) Seterusnya, dengan menggunakan paksi-paksi yang sama, lakarkan satu graf yang sesuai untuk mencari
bilangan penyelesaian kepada persamaan 6 tan x = 2 – x for 0  x  2.
Nyatakan bilangan penyelesaian itu.
Hence, using the same axes, sketch a suitable straight line to find the number of solutions for the equation
6 tan x = 2 – x for 0  x  2. State the number of solutions.
Answer Space / Ruangan Jawapan [(b) y = 1 – , 2]
(a) Amplitude = 1 Shape = 4regions and cycle half regions in 2.
y

1  2 x

(b) From 6 tan x = 2 – x ---------(1) From y = 3 tan x -----------(2)


Substitute (2) into (1): 2y = 2 – x
y=1–
x 0 2
y 1 0
There are 2 solutions
49
9 (a) Lakarkan graf bagi y = – 3 sin 2x untuk 0  x  2 / Sketch the graph of y = – 3 sin 2x for 0  x  2.
(b) Seterusnya, cari nilai / nilai-nilai / julat bagi k dengan keadaan bahawa persamaan 3 sin 2x + k = 1 untuk
0  x   mempunyai
Hence, find the value / values / range of k such that the equation 3 sin 2x + k = 1 for 0  x   has [KBAT]
(i) tiada penyelesaian / no solution,
(ii) hanya satu penyelesaian / only one solution,
(iii) hanya dua penyelesaian / only two solutions,
(iv) hanya tiga penyelesaian / only three solutions.
Answer Space / Ruangan Jawapan [(b)(i) k < – 2 or k > 4, (b)(ii) -2, 4, (b)(iii) −2 < k < 1; 1 < k < 4 @ −2 < k < 4, k  1, (b)(iv) 1]
(a) Amplitude = 3 Shape = wave and cycle 2waves in 2. [Reverse the graph]
y

0  2 x

-3
(b) From 3 sin 2x + k = 1 ---------(1) From y = – 3sin 2x -----------(2)
Substitute (2) into (1): –y+k=1
y=k–1
(i) If k – 1 > 3, the line will not intersect the trigonometric graph
k>4
If k – 1 < – 3, the line will not intersect the trigonometric graph
k<–2
So, there is no solution if k > 4 or k < – 2
(ii) If k – 1 = 3, the line will intersect the trigonometric graph at one point
k=4
If k – 1 = – 3, the line will intersect the trigonometric graph at one point
k=–2
So, there is one solution if k = 4 or k = – 2
(iii) If 0 < k – 1 < 3, the line will intersect the trigonometric graph at two points
1<k<4
If 0 < k – 1 < – 3, the line will intersect the trigonometric graph at one point
1<k <–2
So, there are two solutions if 1 < k < 4 or 1 < k < – 2
(iv) If k – 1 = 0, the line will intersect the trigonometric graph at three points
k=1
So, there are three solutions if k = 1
10 Rajah 10 menunjukkan graf bagi y = a sin ( ) + c, dengan keadaan a, b dan c ialah integer untuk 0 ≤ x ≤ 4 .
Diagram 10 shows a graph of y = a sin ( ) + c, such that a, b and c are integers for 0 ≤ x ≤ 4 . [KBAT]

–5
Rajah / Diagram 10
Graf tersebut melalui 0, – 2 mempunyai titik minimum di 3, – 5 dan titik maksimum di (, 1).
The graph passes through 0, – 2 having a minimum point at 3, – 5 and a maximum point at , 1.
(a) Cari nilai-nilai a, b dan c / Find the values of a, b and c a = 3, b = ½ , c = 2
(b) Seterusnya, lakarkan graf bagi y =  a sin ( ) + c  untuk 0 ≤ x ≤ 4 .
Hence, sketch the graph for y =  a sin ( ) + c  for 0 ≤ x ≤ 4 .
50
KERTAS 2 BAHAGIAN B/A: GEOMETRI KOORDINAT
PAPER 2 SECTION B/A: COORDINATE GEOMETRY
1 Ingat kemahiran asas…
Remember the basic skill…
Rajah bawah menunjukkan dua garis lurus PQ dan QR yang bersilang pada titik Q. Garis lurus PQ adalah
berserenjang dengan garis lurus QR. S ialah satu titik pada cartesan dan titik T membahagi garis lurus QR dalam
nisbah 2 : 3.
Diagram below shows two straight lines PQ and QR intersecting at point Q. The straight line PQ is perpendicular
to the straight line QR. S is a point on the cartesian and the point T divided the line QR in ratio of 2 : 3.

(a) Cari panjang garis PQ. (b) Cari nilai a / Find the value of a.
Find the length of line PQ. Gradient PQ: mPQ =
Let (– 4, 3) = (x1, y1) and (2, 5) = (x2, y2)
=
Length of PQ =
= Gradient QR: mQR =
= =
= PQ is perpendicular to QR: m1m2 = – 1
=  =–1
=2 unit a=–4

(c) Cari lokus S jika ia sentiasa 2 unit dari P. (d) Cari titik T / Find the point T.
Find the locus S if it is always 2 units from P. Q(2, 5) = (x1, y1)
Use distance SP = 2 and let point S = (x, y) R(5, – 4) = (x2, y2)
=2 Ratio 2 : 3 = m : n
=2 Point T = ( , )
Taking square for both sides:
=( , )
(x + 4)2 + (y – 3)2 = 4
(x + 4)(x + 4) + (y – 3)(y – 3) = 4 =( , )
x2 + 4x + 4x + 16 + y2 – 3y – 3y + 9 – 4 = 0
x2 + 8x + y2 – 6y + 21 = 0
x2 + y2 + 8x – 6y + 21 = 0
(e) Cari persamaan garis lurus PR. (f) Cari luas segitiga PQR.
Find the equation of straight line PR. Find the area of triangle PQR.
P(– 4, 3) P(– 4, 3)
R(5, – 4) Q(2, 5)
Gradient PR: m = R(5, – 4)

=– Lawan jam P R Q P
Equation is y – y1 = m(x – x1)
Area =  
y – 3 = – [x – (–4)]
y – 3 = – (x + 4) =  (16 + 25 + 6) – (10 – 8 - 20)

y–3=– x– =  47 – (– 18)

y=– x– +3 =  65 
= 32 unit2
y=– x–
y = – (7x + 1)

51
1 Rajah 1 menunjukkan garis lurus CD yang bertemu dengan garis lurus AB di titik D. Titik C terletak pada paksi-y.
Diagram 1 shows the straight line CD intersect the straight line AB at point D. Point C is on the y-axis.

Rajah / Diagram 1
(a) Tuliskan persamaan AB dalam bentuk pintasan.
Write the equation AB in the intercept form.
(b) Diberi 2AD = DB, carikan koordinat D.
Given 2AD = DB, find the coordinates of D.
(c) Diberi CD berserenjang dengan AB, carikan pintasan-y bagi CD.
Given CD is perpendicular to the AB, find the y-intercept for CD.
Answer Space / Ruangan Jawapan [(a) – = 1, (b) (3, – 4), (c) ]
(a) Equation in intercept form: + =1
+ =1
– =1
(b) Point A(0, – 6) = (x1, y1)
Point B(9, 0) = (x2, y2)
Ratio m : n = 1 : 2
Point D = ( , )
=( , )
= (3, – 4)

(c) mAB =
=

As CD being perpendicular to AB, then mAB mCD = – 1


mCD = – 1
mCD = –

Point C(0, y), Point D(3, – 4)


From mCD = –
=–
y=

52
2 Penyelesaian secara lukisan berskala tidak diterima.
Solution by scale drawing is not accepted.
Rajah 2 menunjukkan garis lurus AC yang bersilang dengan paksi-y pada titik B.
Diagram 2 shows the straight line AC intercepting the y-axis at point B.

Rajah / Diagram 2
Diberi bahawa persamaan garis lurus AC ialah 3y = 2x − 15. Cari
Given the equation of straight line AC is 3y = 2x − 15. Find
(a) persamaan garis lurus yang melalui titik A dan berserenjang dengan AC.
equation of straight line passing through the point A and perpendicular to the AC.
(b) (i) koordinat B,
coordinates of B,
(ii) koordinat C, diberi AB : BC = 2 : 7.
coordinates of C, given AB : BC = 2 : 7.
Answer Space / Ruangan Jawapan [(a) y = – x– , (b)(i) (0, – 5), (b)(ii) ( , 2)
(a) From equation of straight line AC: 3y = 2x − 15
y= x–5
Gradient AC: mAC =
Normal gradient: mAC m2 = – 1
m2 = – 1
m2 = –
Equation with gradient – and passes point A(– 3, – 7): y – (– 7) = – [x – (– 3)]
y+7=– x–
y=– x–

(b)(i) From equation AC: 3y = 2x − 15


At point B, x = 0. 3y = 2(0) − 15
y=−5
So, the point B is (0, – 5)

(b)(i) A(– 3, – 7), B(0, – 5), C(x2, y2), m : n = 2 : 7


Then, B(0, – 5) = ( , )
=( , )

So, = 0 AND =–5


x2 = y2 = 2
Point C = ( , 2)

53
3 Penyelesaian secara lukisan berskala tidak diterima.
Solution by scale drawing is not accepted.
Rajah 3 menunjukkan suatu garis lurus AB.
Diagram 3 shows a straight line AB.

Rajah / Diagram 3
(a) Hitung luas segitiga AOB.
Calculate the area of triangle AOB.
(b) Titik C membahagi dalam garis lurus AB dengan nisbah AC : CB = 3 : 2. Cari koordinat C.
Point C divides the straight line AB with the ratio AC : CB = 3 : 2. Find the coordinates of C.
(c) Titik P bergerak dengan keadaan jaraknya dari A adalah sentiasa dua kali jaraknya dari B.
Cari persamaan lokus bagi P.
Point P is moving such that the distance from A is always two times its distance from B.
Find the locus of P.
Answer Space / Ruangan Jawapan [(a) 9, (b) ( , ), (c) x2 + y2 − 18x + 8y + 45 = 0]

(a) Area of triangle AOB =  


= [(0 + 0 + 24 – (0 + 0 + 6)]
= 9 unit2

(b) A(– 3, 4), C(x, y), B(6, – 2), m : n = 3 : 2


Point C = ( , )
=( , )
=( , )

(c) Point P(x, y), A(– 3, 4), B(6, – 2)


Distance PA = 2  Distance PB
=2
=2
Taking square for both sides: (x + 3)2 + (y – 4)2 = 4[(x – 6)2 + (y + 2)2]
x2 + 6x + 9 + y2 – 8y + 16 = 4[x2 – 12x + 36 + y2 + 4y + 4]
x2 + 6x + y2 – 8y + 25 = 4[x2 – 12x + y2 + 4y + 40]
x2 + 6x + y2 – 8y + 25 = 4x2 – 48x + 4y2 + 16y + 160
3x – 54x + 3y2 + 24y + 135 = 0
2

x2 – 18x + y2 + 8y + 45 = 0
x2 + y2 – 18x + 8y + 45 = 0

54
4 Penyelesaian secara lukisan berskala tidak diterima.
Solution by scale drawing is not accepted.
Dalam Rajah 4, ABC ialah sebuah segi tiga dengan ABC = 90.
In Diagram 4, ABC is a triangle with ABC = 90.

Rajah / Diagram 4
Cari / Find
(a) nilai t / value of t, [2 markah / marks]
(b) luas segi tiga ABC / area of triangle ABC, [2 markah / marks]
(c) jarak berserenjang dari B ke AC, [3 markah / marks]
normal distance from B to AC,
(d) persamaan garis lurus yang melalui titik B dan berserenjang dengan garis lurus AC. [3 markah / marks]
equation of straight line passing through B and perpendicular to the line AC.
Answer Space / Ruangan Jawapan [(a) 8, (b) 6, (c) 2.353, (d) y = −5x + 20]
(a) Gradient AB: mAB = Gradient BC: mBC =
mAB = – 1 mBC =
Since ABC = 90, then mAB mBC = – 1
(– 1)( ) = – 1
( )=1
t=8

(b) Area of triangle ABC =  


= [(5 + 24 + 42) – (21 + 30 + 8)]
= 6 unit2

(d) Gradient AC: mAC =


=
Normal gradient from B to AC = Let m2
So, mBC m2 = – 1
m2 = – 1
m2 = – 5
Let P(x, y) = point at line AC where the point P is normal to B
Then, gradient PB = – 5
=–5
y – 5 = – 5(x – 3)
y – 5 = – 5x + 15
y = – 5x + 20

(c) A(1, 7), C(6, 8), mAC = Substitute (2) into (1): – 5x + 20 – 7 = (x – 1)
Equation AC: y – y1 = m(x – x1) x=
y – 7 = (x – 1) ------(1) From (2): When x = , y = – 5( ) + 20
Equation PB: y = – 5x + 20 ------(2) y=

So, normal distance from point ( , ) to point (3, 5) =


= 2.3534 unit2
55
5 Penyelesaian secara lukisan berskala tidak diterima.
Solution by scale drawing is not accepted.
Dalam Rajah 5, P (−4, 10), Q (0, 6) and R (−2, 2) masing-masing ialah titik tengah bagi garis lurus AB, BC dan AC
supaya ARQP membentuk sebuah segi empat selari.
In Diagram 5, P (−4, 10), Q (0, 6) and R (−2, 2) are the midpoints of straight line AB, BC dan AC such that ARQP
forms a paralellogram.

Rajah / Diagram 5
(a) Cari / Find
(i) persamaan garis lurus AB, [2 markah / marks]
equation of straight line AB,
(ii) persamaan pembahagi dua sama serenjang AC. (Jwp : y = x + 4) [2 markah / marks]
equation of perpendicular bisector AC.
(b) Garis lurus AB diperpanjangkan sehingga bersilang dengan pembahagi dua sama serenjang garis lurus AC di
titik S. Cari koordinat S. [3 markah / marks]
The straight line AB is extended such that it intersects the perpendicular bisector AC at point S. Find the
coordinates of S.
(c) Hitung luas segi tiga PQR. Seterusnya, cari luas segi tiga ABC. [3 markah / marks]
Calculate the area of triangle PQR. Hence, find the area of triangle ABC.
Ruangan Jawapan / Answer Space [(a)(i) y = 2x + 18, (a)(ii) y = − x + 4, (b) (−14, −10), (c) 12, 48]
(a)(i) Gradient RQ: mRQ =
=2
AB is parallel to RQ, so mAB = 2
Equation of AB passing P(– 4, 10) with gradient 2: y – y1 = m(x – x1)
y – 10 = 2[x – (– 4)]
y – 10 = 2x + 8
y = 2x + 18
(a)(ii) Gradient AR = Gradient PQ: mPQ =
=–1
Gradient AR = – 1
For normal gradient AR: mAC mAR = – 1
mAC (– 1) = – 1
mAC = 1
Equation passing R(– 2, 2) with gradient 1: y – y1 = m(x – x1)
y – 2 = 1[x – (– 2)]
y–2=x+2
y=x+4
(b) Equation AB: y = 2x + 18 ---------(1)
Equation of perpendicular bisector AC: y = x + 4 --------------(2)
Equate (1) and (2): 2x + 18 = x + 4
x = – 14
From (1): When x = – 14, y = 2(– 14) + 18
= – 10
So, point S = (– 14, – 10)

(c) Area of triangle PQR =   Area of triangle ABC = 4  Area of triangle PQR
= [(– 8 – 12 + 0) – (– 20 + 0 – 24)] = 48 units2
= 12 unit2
56
6 Rajah 6 menunjukkan sebuah satelit angkasa, S menerima isyarat daripada sebuah parabola, P(-94,46) di bumi.
Kemudian isyarat tersebut dipantulkan kepada parabola, Q. Sudut di antara isyarat yang diterima dan dipantulkan
oleh satelit adalah bersudut tegak. Terdapat sebuah dron di R(-14,6) membahagi kedua-dua parabola tersebut
dengan nisbah 3PR : 20RQ.
Diagram 4 shows a space satellite, S receiving a signal from a parabola, P (-94, 46) on earth. Then the signal is
reflected to a parabola, Q. The angle between the signal received and reflected by the satellite is at right angles.
There is a drone at R(-14, 6) dividing the two parabolas by the ratio 3PR: 20RQ.

Rajah / Diagram 6
Laluan dron ke udara melepasi puncak parabola Q dan mendarat di bumi membentuk separuh bulatan dan
memotong isyarat dari satelit S semasa ketinggian maksimum dron itu terbang, M. Dengan anggapan permukaan
bumi rata dan pusat bagi laluan dron di T(-6, 2).
The drone’s path into the air passes top of the parabola Q and lands on the earth forming a semicircle and the
drone path cross signal from satellite S during the maximum altitude of the drone flying, M. Assuming the earth's
surface is flat and centre of drone path at T(-6, 2).
Cari / Find
(a) koordinat Q / coordinates Q,
(b) koordinat bagi satelit S / satellite S coordinates,
(c) luas kawasan yang dibatasi oleh satelit dan kedua-dua parabola,
the area bounded by the satellite and the two parabolas,
(d) lokus dron R yang terbang tetapi hilang kawalan ke kiri atau ke kanan dari awal penerbangannya.
the locus of an R drone that flies but loses control to the left or to the right from the start of its flight.
Ruangan Jawapan / Answer Space [(a) (– 2, 0), (b) (– 2, 46), (c) 2116, (d) 391x2 + 391y2 – 92x + 1152y - 114788]
(a) P(– 94, 46) = (x1, y1), Q(x2, y2), Ratio m : n = 20 : 3
Point R is a divider
Then, R(– 14, 6) = ( , )
=( , )
= – 14 and =6
x2 = – 2 y2 = 0
So, point Q = (– 2, 0)
(b) Signal from P is directed to S and reflected to Q with angle of 90
Point S = (– 2, 46)

(c) Area of triangle PQS =  


= [(0 – 92 – 92) – (– 92 + 0 – 4324)]
= 2116 unit2
(d) Locus that loses control to the left or to the right from the start of its flight
P(– 94, 46), Q(– 2, 0) R(x, y)
Given the dron divides PQ to 3PR = 20RQ
3 = 20
9 [(x + 94)2 + (y – 46)2 = 400 [(x + 2)2 + y2]
9[x2 + 188x + 8836 + y2 – 92y + 2116] = 400[x2 + 4x + 4 + y2]
9x2 + 1692x + 79524 + 9y2 – 828y + 19044 = 400x2 + 1600x + 1600 + 400y2]
0 = 391x2 + 391y2 – 92x + 828y – 96968
57
KERTAS 2 BAHAGIAN B/A: VEKTOR
PAPER 2 SECTION B/A: VECTOR
Konsep vektor tidak sifar dan tidak selari / Concept of non-zero and non-parallel vector
Diberi vektor-vektor x dan y adalah tidak sifar dan tidak selari dengan keadaan (3p – 2)x = (2q + 6p)y, cari nilai bagi q
dan p / Given vectors x and y are non-zero and non-parallel such that (3p – 2)x = (2q + 6p)y, find the value of q and of p.
Penyelesaian
(3p – 2) = 0 and (2q + 6p) = 0
3p = 2 2q + 6( ) = 0
p= q=–2

Cuba ini / Try this


Vektor a dan vektor b adalah tidak sifar dan tidak selari. Diberi bahawa (h + k)a = (h – k + 1)b, dengan keadaan h dan k
adalah pemalar. Cari nilai bagi h dan k / Vector a and vector b are non-zero and non-parallel. Given that (h + k)a = (h – k
+ 1)b where h and k are constants. Find the value of h and k. [Ans: h = – ½, k = ½]
h + k = 0 ------(1) h–k+1 =0 (1) + (2): 2h = – 1 From (1): When h = – ½ , – ½ + k = 0
h – k = – 1 ------(2) h=–½ k=½

Konsep vektor unit / Concept of unit vector


Magnitud bagi vektor itu ialah 1. Cari magnitude, gunalah konsep teorem pithagoras!
Given that – i + kj is a unit vector, find the value of k / Diberi bahawa – i + kj ialah satu vektor unit, cari nilai bagi k.
Penyelesaian
=1
Ambil kuasa pada kedua-dua belah: (– )2 + k2 = 12
+ k2 = 1
k2 =

k=

=
Konsep vektor unit dalam arah vektor itu, / Concept of unit vector in direction of that vector,
Diberi satu titik A(4, 3), cari unit vektor dalam arah vektor dan dalam bentuk i dan j.
Given the point A(4, 3), find the vector unit in the direction of vector and in form of component i and j.
Penyelesaian
Vektor = 4i + 3j
Magnitude vektor =
=5
Maka, unit vektor dalam arah vektor =
= i+ j
Cuba ini / Try this
Diberi = , cari nilai-nilai yang mungkin bagi p / Given = , find the possible values of p. [Ans: 3]
From = , = i+ j
Since is a unit vector, then ( )2 + ( )2 = 1
+ =1
=
2
p =9
p=3

58
Konsep vektor selari atau vektor segaris / Concept of parallel vector or collinear
Jika vektor p adalah selari dengan q, maka p = mq dengan keadaan m ialah satu pemalar.
If vector p is parallel to q, then p = mq, where m is a constant.
Jika vektor p adalah segaris dengan q, maka p = mq dengan keadaan m ialah satu pemalar.
If vector p is collinear to q, then p = mq, where m is a constant.

Diberi bahawa = 3ka – 4b dan = 4a + 8b. Jika adalah selari dengan , cari nilai bagi k.
Given = 3ka – 4b and = 4a + 8b. If is parallel to , find the value of k.
Penyelesaian
Untuk vektor selari: =m
3ka – 4b = m(4a + 8b)
3ka – 4b = 4ma + 8mb
Bandingkan vektor a untuk kedua-dua belah: 3k = 4m ---------(1)
Bandingkan vektor b untuk kedua-dua belah: – 4 = 8m
m=–½
Ganti m = – ½ ke dalam (1): 3k = 4(– ½)
k=–

Cuba ini / Try this


Diberi bahawa r = 2i – 5j dan s = mi – 3j, cari nilai bagi m jika (2s – r) adalah selari dengan paksi-y.
Given r = 2i – 5j and s = mi – 3j, find the value of m if (2s – r) is parallel to the y-axis. [Ans: m = 1]
Find (2s – r) = 2(mi – 3j) – (2i – 5j)
= 2mi – 6j – 2i + 5j
= (2m – 2)i – j
If (2s – r) is parallel to the y-axis, then (2m – 2) = 0
m=1

Penambahan dan penolakan vektor / Addition and subtraction of vector


Dalam rajah bawah, = a dan = 4b, Diberi Q ialah satu titik pada AC dengan keadaan AQ : QC = 3 : 1, cari
In diagram below, = a and = 4b, Given Q is a point on AC such that AQ : QC = 3 : 1, find the

(a) vektor / vector of ,


(b) vektor / vector of ,
Penyelesaian
(a) = +
= – a + 4b

(b) Given AQ : QC = 3 : 1, then =


= (4b)
=b
= +
=–a+b

59
1 Dalam Rajah 1, OAC dan OBD adalah garis lurus dengan AD dan CB bersilang di P.
In Diagram 1, OAC and OBD are straight lines with AD and CB intersects at P.

Diagram 1
Diberi bahawa =3 , B ialah titik tengah bagi OD dengan keadaan = 3x dan = 5y.
Given that =3 , B is the midpoint of OD where = 3x and = 5y.
(a) Ungkapkan dalam sebutan x dan/ atau y / Express in terms of x and/or y:
(i) ,
(ii),
(b) Diberi bahawa =m dan =n , nyatakan dalam sebutan-sebutan
Given that =m and =n , state in terms of
(i) m, x and y,
(ii) n, x and y,
(c) Seterusnya, cari nilai bagi m dan n.
Hence, find the value of m and n.
Ruangan Jawapan / Answer Space [(a)(i) – 3x + 10y, (a)(ii) 9x – 5y, (b)(i) (3 – 3m)x + 10my, (b)(ii) 9nx + (5 – 5n)y, (c) m = , n = ]

(a)(i) = +
= – 3x + 2(5y)
= – 3x + 10y
(a)(ii) = +
= – 5y + 3(3x)
= 9x – 5y
(b)(i) = +
= +m
= 3x + m(– 3x + 10y)
= (3 – 3m)x + 10my
(b)(ii) = +
= +n
= 5y + n(9x – 5y)
= 9nx + (5 – 5n)y
(c) From (b)(i) and (b)(ii): (3 – 3m)x + 10my = 9nx + (5 – 5n)y
Comparing the vector x: 3 – 3m = 9n
n= --------------(1)
Comparing the vector y: 10m = 5 – 5n ----------(2)

Substitute (1) into (2): 10m = 5 – 5( )


10m × 9 = 5 × 9 – 5( )×9
90m = 45 – 5(3 – 3m)
90m = 45 – 15 + 15m
m=
From (2): 10( ) = 5 – 5n
n=

60
2 Diagram 2 shows a triangle ABC.
Rajah 2 menunjukkan sebuah segitiga ABC.

Diagram 2 / Rajah 2
Given that = x and = y, P and T is the midpoint BC and AC respectively and AP = 3RP.
Diberi bahawa = x dan = y, P dan T ialah titik tengah BC dan AC masing-masing dan AP = 3RP.
(a) Express in terms of x and/or y.
Ungkapkan dalam sebutan x dan/atau y. [2 marks / markah]
(b) Express in terms of x and/or y.
Ungkapkan dalam sebutan x dan/atau y.
(i) ,
(ii)
. [3 marks / markah]
(c) Hence, determine whether the points B, R and T are collinear.
Seterusnya, tentukan sama ada titik-titik B, R dan Tadalah segaris. [5 marks / markah]
Ruangan Jawapan / Answer Space [(a) – x + 2y, (b)(i) 2y, (b)(ii) x – y , (c) not]
(a) = +
= – x + 2y
(b)(i) =2
= 2y
(b)(ii) = +
=–y+
=–y+ ( + )
= – y + (– y + x)
= x– y
(c) = +
= 2y + x – y
= x– y

= +
= +
= (– y + x) + (– x + 2y)
= x+ y
Since m , then the points B, R and T are not collinear.

61
3 Diagram 5 shows a triangle PQR. The point S lies on PQ and the point T lies on QR. The straight line PT intersects
the straight line RS at the point U.
Rajah 5 menunjukkan segi tiga PQR. Titik S terletak pada PQ dan titik T terletak pada QR. Garis lurus PT bersilang dengan
garis lurus RS pada titik U.

Diagram 5 / Rajah 5
→ → → 2 → → 2 →
It is given that PQ = x, PR = y, PS = 3 PQ and QT = 3 QR.
~ ~
→ → → 2 → → 2 →
Diberi bahawa PQ = x, PR = y, PS = PQ dan QT = QR.
~ ~ 3 3
(a) Express in terms of x and/or y:
Ungkapkan dalam sebutan x dan/atau y:
(i) ,

(ii) . [4 marks / markah]

(b) Given that =h and =k , where h and k are constants. State in terms of
Diberi bahawa =h dan =k , dengan keadaan h dan k adalah pemalar. Nyatakan dalam sebutan
(i) h, x and y,

(ii) k, x and y, [3 marks / markah]

(c) Hence, find the value of h and k.


Seterusnya, cari nilai h dan k. [3 marks / markah]
Ruangan Jawapan / Answer Space [(a) x – y, (a)(ii) x + y, (b)(i) h x + h y, (b)(ii) x + (1 – k) y, (c) k = , h = ]
(a)(i) = +
= – y + (x)
(c) From (b)(i): = hx+ hy
= x–y
From (b)(ii): = x + (1 – k) y
(a)(ii) = +
=x+ Comparing vector x: h=
=x+ ( + ) h = 2k -------------(1)

= x + (– x + y) Comparing vector y: h = 1 – k ----------(2)

= x+ y Substitute (1) into (2): (2k) = 1 – k

(b)(i) From =h k=

= h ( x + y) Substitute k = into (1): h = 2( )


= hx+ hy =
(b)(ii) = +
= +k
= y + k ( x – y)
= x + (1 – k) y

62
4 Rajah 4 menunjukkan sebuah segiempat ABCD di mana AED dan EFC adalah garis-garis lurus. Diberi bahawa
= 20v, = 32u, = – 24u + 25v, = dan = .
Diagram 7 shows a quadrilateral ABCD where AED and EFC are straight lines. It is given that = 20v, =
32u, = – 24u + 25v, = and = .

Rajah / Diagram 4
(a) Ungkapkan dalam sebutan u dan v,
Express in terms of u and v,
(i) ,
(ii) ,
(b) Tunjukkan bahawa titik-titik B, F dan D adalah segaris.
Show that the points B, F and D are collinear.
Ruangan Jawapan / Answer Space [(a)(i) 32u – 20v, (a)(ii) 8u – 5v, ]
(a)(i) = +
= – 20v + 32u
= 32u – 20v

(a)(ii) = + +
= – 20v + + .
= – 20v + (32u) + ( + )
= – 20v + 8u + [(32u – 8u) + (– 24u + 25v)]
= – 20v + 8u + [24u – 24u + 25v]
= – 20v + 8u + [25v]
= – 20v + 8u + 15v
= 8u – 5v

(b) = 8u – 5v = 32u – 20v


= 4(8u – 5v)
=4
Since =4 , then B, F and D are collinear

63
5 Sidi berenang arah ke Timur dengan kelajuan 1.5 m s-1 menyeberangi sungai yang lebarnya 600 meter dan mengalir
arah ke utara dengan kelajuan 2.1 m s-1.
Sidi is swimming east at 1.5 m s-1 across a river that is 600 meters wide and flows north at 2.1 m s-1 .

Rajah / Diagram 5
(a) Cari magnitud halaju Sidi.
Magnitude of velocity of Sidi.
(b) Berapa lamakah Sidi mengambil masa untuk menyeberangi sungai itu?
How long will it take Sidi to cross the river?
(c) Berapa jauhkan dia tersasar ketika dia sampai ke seberang sungai?
How far downstream when he reaches the other side of the river
Ruangan Jawapan / Answer Space [(a) 2.58 ms-1, (b) 400 s, (c) 840 m]

600 m

2.1 ms-1 Resultant direction


of Sidi swimming
1.5 ms-1

(a) Magnitude of velocity of Sidi = [Own swimming speed plus overcoming the water stream]
= 2.5817 ms-1

(b) From Velocity =


1.5 =
Expected time = 400 s

(c) How far downstream due water = speed of water  swimming time
= 2.1  400
= 840 m

64
MASTERING IN: CIRCULAR MEASURES
KUASAI: SUKATAN MEMBULAT
1 Tukar sudut darjah kepada unit radian / Converting angle in degree to unit of radian
Berapa radian =  berapa sudut
Contoh: Tukarkan 45 kepada unit radian
45 =  45
= 0.7857 rad @ rad

Tukar radian kepada unit darjah / Converting radian to unit of degree


Berapa degree =  berapa radian
Contoh: Tukarkan 1.5 rad kepada unit degree
1.5 rad =  1.5
= 85.93

Untuk garis lurus, sudut ialah 180 atau  rad atau 3.4142 rad
180 or 3.142 rad

Panjang lengkok, s = r  , dengan keadaan  mesti dalam unit radian


Length of arc, s = r  where  must be in unit radian
P
r Length of arc PQ = r


O Q
Luas sektor POQ = r  where  must be in unit radian
2

P
Tembereng PQ
Segment PQ
O 
Q
Luas tembereng = r ( – sin )
2

mesti dalam radian mesti dalam sudut darjah

Imbas kembali: teorem Pithagoras & Formula Trigonometri

a2 + b2 = c2 sin  = cos  = tan  =

65
1 Diagram 1 shows a sector AOB of a circle with centre O. The point P lies on OA, the point Q lies on OB and PQ is
perpendicular to OB. The length of OP is 9 cm and  AOB = rad.
Rajah 1 menunjukkan sektor AOB bagi sebuah bulatan, berpusat di O. Titik P terletak di atas OA, titik Q terletak di atas OB
dan PQ adalah berserenjang pada garis OB. Panjang OP ialah 9 cm dan  AOB = rad.

Diagram 1 / Rajah 1
Diberi bahawa OP : OA = 3 : 5.
It is given that OP: OA = 3 : 5.
[Using / Guna  = 3.142]
Calculate
Hitung
(a) the length, in cm, of PA,
panjang, dalam cm, PA, [1 mark / markah]

(b) the perimeter, in cm, of the shaded region,


perimeter, dalam cm, kawasan berlorek, [1 mark / markah]

(c) the area, in cm2, of the shaded region.


luas, dalam cm2, kawasan berlorek. [1 mark / markah]
Ruangan Jawapan / Answer Space [(a) 6, (b) 25.5608 cm, (c) 866.0356 cm2]
(a) Given OP : OA = 3 : 5
9 : OA = 3 : 5
=
OA = 15 cm
Length of PA = 15 cm – 9cm
= 6 cm
(b) cos 30º = sin 30º =
cos 30º = sin 30º =
OQ = 7.7942 cm QP = 4.5 cm

Length of QB = 15 cm – 7.7942 cm length of arc AB = rθ


= 7.2058 cm = 15(
= 7.885 cm
So, the perimeter of the shaded region = AP + PQ + QB + arc BA
= 6 cm + 4.5 cm + 7.2058 cm + 7.885 cm
= 25.5608 cm
(c) The area of the shaded region = Area of sector – Area of triangle
= ½ r2 θ – ½ × OQ × QP
= ½ (15)2( ) – ½ × 7.7942 × 4.5
= 866.0356 cm2

66
2 In Diagram 2, PBQ is a semicircle with centre O and has a radius of 10 m. RAQ is a sector of a circle with centre A
and has a radius of 16 m.
Dalam Rajah 2, PBQ ialah separuh bulatan berpusat di O dan mempunyai jejari 10m. RAQ ialah sektor bulatan berpusat di A
dan mempunyai jejari 16 m.

Diagram 2 / Rajah 2
It is given that AB = 10 m and  BOQ = 1.876 radians. [Use  = 3.142]
Diberi bahawa AB = 10 m dan  BOQ = 1.876 radian. [Guna  = 3.142]
Calculate / Hitung
(a) the area, in m2, of sector BOQ,
luas, dalam m2, sektor BOQ, [2 marks / markah]

(b) the perimeter, in m, of the shaded region,


perimeter, dalam m, kawasan berlorek, [4 marks / markah]

(c) the area, in m2, of the shaded region.


luas, dalam m2, kawasan berlorek, [4 marks / markah]
Ruangan Jawapan / Answer Space [(a) 93.8 cm2, (b) 45.016 cm, (c) 39.6121 cm2]
(a) The area of sector BOQ = ½ (10)2(1.876)
= 93.8 cm2
(b) Length of arc BQ = 10(1.876)
= 18.76 cm

BAO = BOA = 180º – BOQ


= 180º – ( )× 1.876
= 72.53º
= × 72.53
= 1.266 rad
Length of arc RQ = 16(1.266)
= 20.256 cm
Length of BR = 16 cm – 10 cm
= 6 cm
So, the perimeter of the shaded region = RB + arc BQ + arc QR
= 6 cm + 18.76 + 20.256 cm
= 45.016 cm
(c) Area of sector ARQ = ½ (16)2(1.266)
= 162.048 cm2
Area of sector OBQ = ½ (10)2(1.876)
= 93.80 cm2

Area of triangles ABO = ½ (10)(10)sin 34.94º


= 28.6359 cm2

So, the area of the shaded region = Area of sector ARQ – Area of sector OBQ – Area of triangle ABO
= 162.048cm2 – 93.80 cm2 – 28.6359 cm2
= 39.6121 cm2
67
3 Diagram 3 shows a circular lake with centre O. Given that diameter GH= 24 m and the length of arc HK= 15 m.
Point J is right above H. Encik Kamal owns a villa in the shaded region HJK. The segment of GK is used as fish
rearing farm.
Rajah 3 menunjukkan sebuah tasik berbentuk bulatan yang berpusat di O. Diberi bahawa diameter GH = 24 m dan panjang
lengkok HK = 15 m. Titik J berada tegak di atas H. Encik Kamal memiliki sebuah villa di kawasan berlorek HJK. Segmen GK
digunakan sebagai tempat perternakan ikan.

Diagram 3 / Rajah 3
(a) Show that α = 0.625 radian / Tunjukkan bahawa α = 0.625 radian. [2 marks / markah]
(b) Calculate the perimeter of fish farm GK / Cari perimeter tempat ternakan ikan GK. [4 marks / markah]
(c) Calculate the area, in m2, of Encik Kamal's villa.
Hitung luas, dalam m2, villa Encil Kamal. [4 marks / markah]
Ruangan Jawapan / Answer Space [(b) 42.1671 cm, (c) 49.4696 cm2]
(a) Radius OH = 12 m
Length of arc KH = 15 m
So, 12θ = 15
θ = 1.25 rad
Then, α = θ
= (1.25)
α = 0.625 radian

(b) Length of arc GK = r θGOK


= 12(3.142 – 1.25 )
= 22.704 cm

Length of GK: GK2 = 122 + 122 – 2(12)(12)cos(180 – × 1.25)


2 2 2
GK = 12 + 12 – 2(12)(12)cos108.40º
GK = 19.4631 cm
So, the perimeter of fish farm GK = 22.704 cm + 19.4631 cm
= 42.1671 cm

(c) Length of JH: tan( × 0.625) =


JH = 17.3156 cm

Area of triangle JGH = × 24 × 17.3156


= 207.7872 cm2

Area of sector KOH = × 122 × 1.25


= 90 cm2

Area of triangle GOK = × 12 × 12 × sin 108.40º


= 68.3176 cm2
So, the area of Encik Kamal's villa KJH = Area of triangle JGH – Area of sector KOH – Area of triangle GOK
= 207.7872 – 90 – 68.3176
= 49.4696 cm2

68
4 Diagram 4 shows PQ and PR are tangent to the circle with centre O. OQR is a sector with centre O and NQR is
another sector with centre N.
Rajah 4 menunjukkan PQ dan PR ialah tangen kepada bulatan berpusat O. OQR ialah sektor bulatan berpusat O dan NQR
ialah sektor bulatan lain yang berpusat N.

Diagram 4 / Rajah 4
It is given that OQ = 8 cm, QN= RN = 12 cm and OP = 17 cm
Diberi bahawa OQ = 8 cm, QN = RN = 12 cm dan OP =17 cm. [Use / Guna π = 3.142]
(a) Show that α = 2.162 (to three decimals places),
Tunjukkan bahawa α = 2.162 (kepada tiga tempat perpuluhan), [2 marks / markah]
(b) Find the perimeter, in cm, of segment NQ.
Cari perimeter, dalam cm, segmen NQ. [4 marks / markah]
(c) Calculate the area, in cm2, of the shaded region.
Hitung luas, dalam cm2, kawasan berlorek. [4 marks / markah]
Ruangan Jawapan / Answer Space [(b) 30.2054 cm, (c) 47.8239 cm2]
(a) cos α =
=
α = 61.93º
α = 123.86º
= × 123.86º
α = 2.162 rad (Shown)
(b) NOQ = 180º – 61.93º
= 118.07º
= × 118.07º
= 2.0607 rad
Length of arc NQ = 8(2.0607)
= 16.4857 cm
Length of NQ: NQ2 = 82 + 82 – 2(8)(8)cos118.07º
NQ = 13.7197 cm
So, the perimeter of segment NQ = 16.4857 cm + 13.7197 cm
= 30.2054 cm
(c) Area of sector QOR = × 82 × 2.162
= 69.184 cm2

Area of triangle NOQ = × 8 × 8 × sin 118.07º


= 28.2359 cm2
Then, total area of triangles NOQ and NOR = 28.2359 cm2 × 2 cm
= 56.4719 cm2

Area of sector QNR = × 122 × (2.162 ÷ 2)


= 77.832 cm2
So, the area of the shaded region = (69.184 + 56.4719) – 77.832
= 47.8239 cm2

69
5 Diagram 5 shows a circle EGDH, centre O and radius 5 cm. EB, DB and AC are tangents to the circle at E, D and G
respectively. The straight lines, OA and OC intersect the circle at E and D respectively. ABC is an arc of a circle,
centre O.
Rajah 5 menunjukkan sebuah bulatan EGDH, berpusat di O dan berjejari 5cm. EB, DB dan AC masing-masing ialah tangen
kepada bulatan di E, D dan G.Garis lurus OA dan OC masing-masing bersilang pada bulatan itu di E dan D. ABC ialah
lengkok bulatan berpusat di O.

Diagram 5 / Rajah 5
Given BD = 12 cm, calculate
Diberi BD = 12 cm, hitung
(a) the angle  , in radians,
sudut  , dalam radian, [3 marks / markah]
(b) the length, in cm, of the arc ABC,
panjang, dalam cm, lengkok ABC, [3 marks / markah]
(c) the area, in cm2, of the shaded region.
luas, dalam cm2, kawasan berlorek. [4 marks / markah]
Ruangan Jawapan / Answer Space [(a) 2.3520 rad, (b) 30.5758 cm, (c) 138.7414 cm2]
(a) tan α =
tan α =
α = 67.38º
α = 134.76º
= × 134.76º
α = 2.3520 rad.

(b) cos α =
cos 67.38º =
OC = 12.9999 cm
So, the length of the arc ABC = 12.9999 (2.3520)
= 30.5758 cm

(c) Area of sector AOCB = × 12.99992 × 2.3520


= 198.7409 cm2
Area of triangle AOCG = 2 × Area of triangle OGC
=2× ×5×
= 59.9995 cm2
So, area of the shaded region = 198.7409 – 59.9995
= 138.7414 cm2

70
6 Rajah 6 menunjukkan pandangan hadapan sebuah alat penimbang berbentuk bulatan dengan jejari 12 cm dengan
pusat , dan disokong di atas tapak berbentuk kuboid. Panjang tapak kawasan berlorek adalah 30 cm dan tingginya
6 cm. Berat maksimum yang boleh di tampung adalah 2.4 kg. Apabila ditimbang paha ayam, jarum timbangan
menunjukkan kedudukan .
Diagram 6 shows a front view of a weighing machine in the form of a circle with radius of 12 cm with center O, and
supported at the site of the cuboid. The length of the shaded area is the site of 30 cm and a height of 6 cm. The
maximum weight that can be at capacity is 2.4 kg. When a chicken thigh is weighed, the scales show the position A.

12 cm
6 cm
T
6 cm

Rajah / Diagram 6
[Gunakan = 3.142] / [Use = 3.142]
Cari/ Find
(a) jisim paha ayam, dalam kg,
the mass of chicken thigh, in kg,
(b) luas, dalam 2 , rantau berlorek.

area, in 2 , of the shaded region.


Ruangan Jawapan / Answer Space [(a) 0.8 kg, (b) 92.53 91.5574 cm2]

(a) cos TOA =


cos TOA = 0.5
TOA = 60
Maximum mass is 2.4 kg for 360
The needle is moving = (180 – 60)
= 120
Then, the mass for 120 =  2.4 kg
= 0.8 kg
(b) Area of rectangular shape = 30 cm  6 cm
= 180 cm2
Area of sector OBCA = r2 
= (12)2 (  120)
= 150.7964 cm2

Area of triangle OTA = (6)(12) sin 60


= 31.1769 cm2
So, area of shaded rectangular shape = 180 – (150.7964 – 31.1769 – 31.1769)
= 91.5574 cm2

71
7 Puan Manisah membeli sebiji kek lapis berpusat seperti ditunjukkan dalam Rajah 7, mempunyai diameter 20 cm
dan ketebalan 8 , untuk harijadi anaknya / Puan Manisah bought a circular layer cake with centre as shown as
Diagram 7, has a diameter 20 cm and thickness of 8 , for his son’s birthday.

Rajah / Diagram 7
Sektor minor dengan sudut 20° dipotong untuk diberikan kepada ibunya.
Minor sector with angle 20° is cut out and give to his mother.
(a) Cari jumlah luas permukaan bahagian kek yang diberi kepada ibunya.
Find the total surface area of the piece of cake given to his mother.
3
(b) Seterusnya, calculate the volume, dalam , baki kek yang tinggal.
3
Hence, calculate the volume, in, of the remaining cake.
(c) Dalam musim pandemik ini, harga kek telah meningkat sebanyak c % kerana terpaksa menampung kos
penghantaran. Jika selama 3 tahun kebelakangan ini, harga kek tidak pernah berubah iaitu kekal dengan harga
RM 120, ungkapkan harga baru kek harijadi tersebut dalam sebutan c
In this pandemic season, cost of the cake has increase by c% because of wanting to cover the delivery services
charge. . If 3 years behind, the cost of the cake was never unchanged, that is maintain RM120, express the
new price of the cake in terms of c.
Ruangan Jawapan / Answer Space [(a) 222.83, (b) , (c) (100 + c)]

(a) Surface area for this sector = r2 


= ( )(10)2 
= 17.4533 cm2
10 cm

10 cm
Surface area for this side Surface area for this side = 80 cm2
= 10 cm  8 cm 8 cm
= 80 cm2
Surface area for this curve side = 2rh 
= 2( )(10)(8) 
Surface area for this sector = 17.4533 cm2 = 27.9253 cm2

So, total surface area of cake = 80 + 80 + 17.4533 + 17.4533 + 27.9253


= 222.8319 cm2

(b) Volume of cake left = r2h 
= (10)2(8) 
= cm3
(c) New price = 120 + 120 

= (100 + c)

= (100 + c)

72
KERTAS 2 BAHAGIAN A/B: PEMBEZAAN & PENGAMIRAN
PAPER 2 SECTION A/B: DIFFERENTIATION & INTEGRATION
Ingat konsep pembezaan / Remember the concept of differentiation
Pembezaan secara terus Pembezaan secara formula hasil darah Pembezaan secara formula bahagi
Direct differentiation Differentiation by product formula Differentiation by division rule
Kes 1: Secara terus Cari bagi y = 2x(4x – 1)3 Cari bagi y =
Cari bagi y = 3x2 – + 1 Ingat: 2x tidak dapat didarab masuk Kaedah 1: Guna cara seperti Kes 4
y = 3x – + 1
2 ke dalam kurungan yang ada kuasa!! Kaedah 2: Guna hukum bahagi
y = 2x(4x – 1)3 Let u = 5 dan v = (x – 3)2
y = 3x2 – 2x – 1 + 1
Biar u = 2x and v = (4x – 1)3 =0 = 2(x – 3)
= 6x 2 – 1 – (– 1)2x – 1 – 1 + 0
=2 = 3(4x – 1)3 – 1 (4)
= 6x + 2x – 2 =
= 12(4x – 1)2
= 6x + =
=u +v
= 2x[12(4x – 1)2] + (4x – 1)3 (2) =
Kes 2: Kembangkan dahulu
= 24x(4x – 1)2 + 2(4x – 1)3
Cari bagi y = (x + 3)(2x – 1) =–
= 2(4x – 1)2 [12x + (4x – 1)]
Kembangkan dahulu = 2(4x – 1)2 (18x – 1)
y = (x + 3)(2x – 1) Cari bagi y =
= 2x2 – x + 6x – 3 Cari bagi y = (x – 1)(2x + 3) 4
Let u = 3x dan v = (x + 2)
y = 2x2 + 5x – 3
Biar u = (x – 1) and v = (2x + 3) 4
=3 =1
= 4x + 5 4–1
=1 = 4(2x + 3) (2)
=
Kes 3: y = a(bx + c) ,n
= na(bx + c) n-1
(b) = 8(2x + 3)3
=
Cari bagi y = 2(3x – 1) 4 =u +v
= (x – 1)[8(2x + 3)3]+(2x + 3)4 (1) =
y = 2(3x – 1)4
= 8(x – 1)(2x + 3)3 + (2x + 3)4
= (4)2(3x – 1)4 – 1 (3) =
= (2x + 3)3 [8(x – 1) + (2x + 3)]
= 24(3x – 1)3 = (2x + 3)3 [8x – 8 + 2x + 3]
= (2x + 3)3 (10x – 5)
Kes 4: Tarik naik sebelum pembezaan
Cari bagi y = Cari bagi y = (x + 5)2
y= Biar u = dan v = (x + 5)2
= (5x + 1) – 2 = = 2(x + 5)2 – 1 (1)
= – 2(5x + 1) – 2 – 1 (5)
= = 2(x + 5)
= – 10(5x + 1) – 3
=
=–
=

=u +v
= [2(x + 5)] + (x + 5)2 ( )

=2 (x + 5) +

73
Perlu ingat bahawa…
Need to remember that…
 ialah kecerunan iaitu m = / is the gradient where m =
 ialah fungsi kecerunan @ tangen kepada lengkungan / is the gradient function or tangent to the curve
= 0 [pada titik maksimum @ titik minimum @ titik pusingan // at maximum point @ minimum point @ turning point]

Fungsi kecerunan kepada lengkungan pada titik (2, 4) ialah 2x + 3. Cari persamaan lengkung itu pada titik (2, 4).
The gradient function to the curve at point (2, 4) is 2x + 3. Find the equation of the curve at point (2, 4).
Gradient function = 2x + 3
= 2x + 3

Integrate: y= + 3x + c [Kamirkan / Integrate: Kuasa tambah 1, dibahagi kuasa baharu]


2
y = x + 3x + c
Pada titik (2, 4): 4 = 22 + 3(2) + c
c=–6
Maka, persamaan lengkung itu ialah y = x2 + 3x – 6

Diberi persamaan lengkung y = x2 + 2ax – 8 mempunyai titik maksimum pada x = 3. Cari nilai a.
Given the equation of curve y = x2 + 2ax – 8 has the maximum point at x = 3. Find the value of a.
y = x2 + 2ax – 8
= 2x + 2a
0 = 2(3) + 2a [Pada titik maksimum, = 0 dan x = 3]
a=–3

Kamiran / Integration
 Petua kamiran: Kuasa tambah 1, kemudian dibahagi dengan kuasa baharu
 Kurungan yang ada kuasa, jangan ubah kurungan itu!
y= y= Cari luas dan isipadu bagi kawasan berlorek.
Find the area and volume for the shaded region.
= + + 3x + c =
y
y = x4 + + 3x + c = +c y=
y= +c
0 4 x
y= y= +c Luas = Diberi y =
= +c = y2 = x
y= +c = Isipadu =
y=
= = =

Find y = = + 5x + c
= =
= y= + 5x + c
y = – 3x – 1 + 5x + c
= = =[ ]–[ ]
= [32 – 3(3)] – [12 – 3(1)] y = – + 5x + c
=2 =[ ]–[ ] = 8 unit3
= unit2

74
1 Diagram 1 shows the parts of the curve y = which passes through S(– 1, 2).
Rajah 1 menunjukkan sebahagian daripada lengkung y = yang melalui S(– 1, 2).

Diagram 1 / Rajah 1
(a) Find the equation of the normal to the curve at point S(– 1, 2).
Cari persamaan normal kepada lengkung itu pada titik S(– 1, 2). [4 marks / markah]

(b) A region is bounded by the curve, the x-axis and the straight line x = – 2 and x = – 3.
Satu rantau dibatasi oleh lengkung, pada paksi-x dan garis lurus x = – 2 and x = – 3.
(i) Find the area of the region.
Cari luas rantau itu.

(ii)The region is revolved through 360º about the x-axis. Find the volume generated, in terms of π.
Rantau itu dikisarkan melalui 360º pada paksi-x. Cari isipadu yang dijanakan, dalam sebutan π .
[6 marks / markah]
Ruangan Jawapan / Answer Space [(a) y = – x + , (b)(i) , (b)(ii) π]

(a) From y = (b)(ii) Volume = π


–2
= 8(3x + 1) =π
–3
= (-2) 8(3x + 1) (3) =π
=
= π[ ]-3-2
At point S( – 1, 2): = = π ([ ]–[ ])
=6 =π[ ]–[ ]
Normal gradient: m1m2 = – 1 = π
6m2 = – 1
m2 = –
Equation of normal to curve at point S(– 1, 2):
y – y1 = m(x – x1)
y – 2 = – [x – ( – 1)]
y – 2 = – (x + 1)
y=– x+
(b)(i) Area of the region =
=[ ]-3-2

=[ ]–[ ]

=( )–( )
=

75
2 Diagram 2 shows a curve y = 1 – x3 intersecting the x-axis at point Q and intersects the y-axis at point P. The curve
also passes through the point R(k, – 7) and S(– 1, 2).
Rajah 2 menunjukkan lengkung y = 1 – x3 yang bersilang dengan paksi-x di titik Q dan bersilang dengan paksy-y di titik P.
Lengkung itu juga melalui titik R(k, – 7) dan S(– 1, 2).

Diagram 2 / Rajah 2
Find / Cari
(a) (i) coordinates of P and Q,
koordinat titik P dan Q,
(ii) value of k.
nilai k. [3 marks / markah]
(b) area of region B,
luas rantau B, [3 marks / markah]
(c) The region A is revolved through 360º about the x-axis. Find the volume generated, in terms of π.
Rantau B itu dikisarkan melalui 360º pada paksi-x. Cari isipadu yang dijanakan, dalam sebutan π .
[4 marks / markah]
Ruangan Jawapan / Answer Space [(a)(i) (1, 0), (a)(ii) 2, (b) , (c) π]
3 3
(a)(i) At point P(0, y): y = 1 – (0) At point Q(x, 0): 0 = 1 – x
=1 x=1
So, coordinate of P(0, 1) So, coordinate of P(1, 0)

(a)(ii) At R(k, – 7): y = 1 – x3


– 7 = 1 – k3
k3 = 8
k=2

(b) From y = 1 – x3
Area of region B =  
=  [x – ]12

=  [2 – ] – [1 – ] 
2
= unit
(c) Volume = π

= π[x – + ]-10

= π([0] – [(– 1) – + ])
= π unit3

76
3 (a) Diagram 3(a) shows a side elevation of a metal cylindrical container without cover. The inner surface of the
container can be represented by quadratic equation y = 2x2 + 1. The height of the container is 9 cm.
Rajah 2(a) menunjukkan pandangan sisi bagi sebuah bekas logam silinder tanpa penutup. Permukaan dalam bagi bekas
itu boleh diwakili oleh persamaan kuadratik y = 2x2 + 1. Tinggi bekas itu ialah 9 cm.

Diagram 2 / Rajah 2
3
Find the volume, in cm , of metal needed to make the container.
Cari isipadu, dalam cm3, logam yang diperlukan untuk membuat bekas itu. [5marks / markah]
(b) Diagram 2(b) shows the straight line 2x – 3y + 6 = 0 intersects the curve y = at point P. Find
Rajah 2(b) menunjukkan garis lurus 2x – 3y + 6 = 0 yang menyilang lengkung y = pada titik P. Cari

Diagram 2(b) / Rajah 2(b)


(i) the value of h / nilai h, [1 mark / markah]
(ii) the area of the shaded region / luas bagi kawasan berlorek. [4 marks / markah]
Ruangan Jawapan / Answer Space [(a) 65π, (b)(i) 3, (b)(i) ]
2
(a) From the quadratic equation y = 2x + 1 (b)(i) From 2x – 3y + 6 = 0 at point P(h, 4)
The minimum point = (0, 1) 2h – 3(4) + 6 = 0
When height is 9, then 9 = 2x2 + 1 h=3
x = 2 or – 2
(b)(ii) From 2x – 3y + 6 = 0
From y = 2x2 + 1 At y-intercept: x = 0. Then, 2(0) – 3y + 6 = 0
2x2 = y – 1 y=2

x2 =
So, area of shaded region = ½ (2 + 4)(3) +
=9+[ ]35

=9+[ ]–[ ]

=9+[ ]–[ ]
Volume of metal needed = πr2h – π
= unit2
= πr2h – π
= π(3)2(9) – π [ ]19

= 81π – π [( )–( )]
= 81π – π [ – ( )]
= 81π – 16π
= 65π unit3
77
4 A manufacturing company produced and sells tables. The cost function is given by C(x) = 1500 – + 65x2, where x
is the number of tables produced. Each table is sold at a price of RM3000.
Sebuah syarikat menghasilkan dan menjual meja. Fungsi kos diberi oleh C(x) = 1500 – + 65x2, dengan keadaan x ialah
bilangan meja yang dihasilkan. Setiap meja dijual pada harga RM3000.
Find
Cari
(a) the profit function, P(x),
fungsi keuntungan, P(x),

(b) the maximum number of tables to be produced to gain a maximum profit,


bilangan maksimum meja yang dihasilkan untuk mendapat keuntungan maksimum ,

(c) the maximum profit that can be obtained.


keuntungan maksimum yang boleh diperolehi.
Ruangan Jawapan / Answer Space [(a) 3000x – 1500 + – 65x2, (b) 30, (c) 3900]
(a) The profit function, P(x) = 3000x – C(x)
= 3000x – (1500 – + 65x2)

= 3000x – 1500 + – 65x2

(b) From P(x) = 3000x – 1500 + – 65x2


= 3000 + x2 – 130x

= 2x – 130

At maximum profit, = 0. Then, 3000 + x2 – 130x = 0


x2 – 130x + 3000 = 0
(x – 100)(x – 30) = 0
x = 100 x = 30

When x = 100, = 2(100) – 130


= 70 (minimum)

When x = 30, = 2(30) – 130


= – 70 (maximum)

So, the maximum number of tables to be produced to gain a maximum profit is 30

(c) When x = 30, Profit P = 3000(30) – 1500 + – 65(30)2


= 39000

78
5 Diagram 5 shows two shaded regions, R and S which are bounded by the line y = 8x, the curve y = 9 – x2 and the x-
axis / Rajah 5 menunjukkan dua kawasan berlorek, R dan S yang dibatasi oleh garis y = 8x, lengkung y = 9 – x2 dan paksi-x.

Diagram 5 / Rajah 5

(a) Find the coordinates of A.


Cari coordinate A. [3 marks / markah]
(b) Calculate / Hitung
(i) the total area of the shaded region,
jumlah luas bagi kawasan berlorek, [3 marks / markah]
(ii) the volume generated, in terms of π, when the shaded region S is rotated through 360º about the x-axis.
isipadu yang dijanakan, dalam sebutan π, apabila kawasan berlorek S diputarkan melalui 360º pada paksi-x.
[4 marks / markah]
Ruangan Jawapan / Answer Space [(a) (1, 8), (b)(i) , (b)(ii) π]
(a) y = 8x ---------------(1)
y = 9 – x2 -------------(2)
(1) into (2): 8x = 9 – x2
2
x + 8x – 9 = 0
(x – 1)(x + 9) = 0
x = 1 x = – 9 (rejected because point A is at positive x)
From (1): When x = 1, y = 8(1)
=8
So, the point A is (1, 8)

(b)(i) Total area of the shaded region = ½ (1)(8) +


= 4 + [9x – ]13
= 4 + [(27 – ) – (9 – )]
2
= unit

(b)(ii) Volume generated = π




= π [81x – 6x3 + ]13

= π [[81(3) – 6(3)3 + ] – [81(1) – 6(1)3 + ]


=π[ – ]
= π unit3

79
6 SPM 2018
Diagram 6 shows the straight line 4y = x – 2 touches the curve x = y2 + 6 at point A.
Rajah 6 menunjukkan garis lurus 4y = x – 2 menyentuh lengkung x = y2 + 6 pada titik A.

Diagram 6 / Rajah 6
Find / Cari
(a) the coordinate of A,
koordinat titik A, [2 marks / markah]

(b) the area of the shaded region, [5 marks / markah]


luas kawasan berlorek,

(c) the volume of revolution, in terms of π, when the region bounded by the curve and the straight line x = 8 is
revolved through 180º about the x-axis.
isipadu kisaran, dalam sebutan π, apabila rantau yang dibatasi oleh lengkung dan garis lurus x = 8 dikisarkan melalui
180º pada paksi-x.
[3 marks / markah]
Ruangan Jawapan / Answer Space [(a) (10, 2), (b) , (c) 2 π]

(a) 4y = x – 2 ---------(1)
x = y2 + 6 -----------(2)
Substitute (2) into (1): 4y = y2 + 6 – 2 From (1): When y = 2, 4(2) = x – 2
y2 – 4y + 4 = 0 x = 10
(y – 2)(y – 2) = 0 So, the point A is (10, 2)
y=2
(b) From 4y = x – 2 For the curve x = y2 + 6
At x-intercept, y = 0. Then 4(0) = x – 2 When y = 0, x = 6
x=2

Area of shaded region = ½ (10 – 2)(2) – dx

=8–[ ]610

=8–([ ]–[ ])
=8–( – 0)
2
= unit
2
(c) From x = y + 6 Volume = π
y2 = x – 6 =π
=π[ ]68

= π ([ ]–π[ ])
= π ([– 16] – [– 18])
= 2 π unit3

80
KUASAI: PERSAMAAN SERENTAK
MASTERING IN: SIMULTANEOUS EQUATION
1 Solve the simultaneous equations.
Selesaikan persamaan serentak berikut.
2y – x = 7
2
x + 3xy + 2 = y
Correct your answer to four decimal places.
Bundarkan jawapan anda betul kepada empat tempat perpuluhan .
Ruangan Jawapan / Answer Space [y = 3.5724 when x = 0.1448 or y = 1.4276 when x = – 4.1448]
From 2y – x = 7
2y – 7 = x
x = 2y – 7 -------(1)
2
From x + 3xy + 2 = y ---------(2)
Substitute (1) into (2): (2y – 7)2 + 3(2y – 7)y + 2 = y
(2y – 7)( 2y – 7) + 3(2y – 7)y + 2 – y = 0
4y – 14y – 14y + 49 + 6y2 – 21y + 2 – y = 0
2

10y2 – 50y + 51 = 0
a = 10, b = – 50, c = 51
Then, y =  must show this step
= 3.5724 or 1.4276

From (1): When y = 3.5724, then x = 2(3.5724) – 7


= 0.1448
When y = 1.4276, then x = 2(1.4276) – 7
= – 4.1448

2 Find the coordinates of points of intersection of the line 2x – y = – 1 with the curve x2 + 2y2 = 3.
Cari koordinat titik persilangan antara garis lurus 2x – y = – 1 dengan lengkung x + 2y = 3.
2 2

Ruangan Jawapan / Answer Space [( , ) and (– 1, – 1)]


From 2x – y = – 1
2x + 1 = y
y = 2x + 1 ---------(1)
From x + 2y2 = 3 --------(2)
2

Substitute (1) into (2): x2 + 2(2x + 1)2 = 3


x2 + 2(2x + 1)(2x + 1) – 3 = 0
x2 + 2[4x2 + 4x + 1] – 3 = 0
x2 + 8x2 + 8x + 2 – 3 = 0
9x2 + 8x – 1 = 0
(9x – 1)(x + 1) = 0  must show this step
x= x=–1
From (1): When x = , then y = 2( ) + 1
=
When x = – 1, then y = 2(– 1) + 1
=–1
So the coordinates are ( , ) and (– 1, – 1)

81
3 Solve the simultaneous equations.
Selesaikan persamaan serentak berikut.
x + 3y + 10 = 0
+ +1=0
Ruangan Jawapan / Answer Space [x = 20 when y = – 10 or x = 2 when y = – 4]
From x + 3y + 10 = 0
x = – 3y – 10 ---------(1)
From + +1=0

Multiply all terms by xy:  xy +  xy + 1  xy = 0


4y + 12x + xy = 0 -----------(2)
Substitute (1) into (2): 4y + 12(– 3y – 10) + (– 3y – 10)y = 0
4y – 36y – 120 – 3y2 – 10y = 0
– 3y2 – 42y – 120 = 0
3y2 + 42y + 120 = 0
y2 + 14y + 40 = 0
(y + 10)(y + 4) = 0
y = – 10 y = – 4
From (1): When y = – 10, then x = – 3(– 10) – 10
= 20
When y = – 4, then x = – 3(– 4) – 10
=2

4 Solve the simultaneous equations.


Selesaikan persamaan serentak berikut.
x–y=3
2
x + 3y = 6
Give your answers correct to 3 decimal places.
Beri jawapan anda betul kepada 3 tempat perpuluhan.
Ruangan Jawapan / Answer Space [x = 2.449 when y = – 0.551 or x = – 2.449 when y = – 5.449]
From x – y = 3
x = y + 3 -------(1)
From x2 + 3y = 6 -------(2)
Substitute (1) into (2): (y + 3)2 + 3y = 6
(y + 3)(y + 3) + 3y – 6 = 0
y2 + 6y + 9 + 3y – 6 = 0
y2 + 9y + 3 = 0
a = 1, b = 9, c = 3
Then, y =  must show this step
= – 0.347 or – 8.653

From (1): When y = – 0.347, then x = – 0.347 + 3


= 2.653
When y = – 8.653, then x = – 8.653 + 3
= – 5.653

82
5 Solve the simultaneous equations 5x + 3y = 3x2 – y2 = 2x + y + 1.
Selesaikan persamaan serentak 5x + 3y = 3x2 – y2 = 2x + y + 1.
Ruangan Jawapan / Answer Space [x = 1 when y = – 1 or x = – when y = 4]
For 5x + 3y = 3x2 – y2 = 2x + y + 1.
Let 5x + 3y = 2x + y + 1 Let 3x2 – y2 = 2x + y + 1
3y – y = 2x + 1 – 5x 3x2 – y2 – 2x – y – 1 = 0 ---------(2)
2 y = 1 – 3x
y= --------(1)
Substitute (1) into (2): 3x2 – ( )2 – 2x – ( )–1=0
3x2 – ( )( ) – 2x – ( )–1=0
Multiply all by 4: 3x  4 – (
2
)( )  4 – 2x  4 – ( )4 –14=0
12x – (1 – 3x)(1 – 3x) – 8x – 2(1 – 3x) – 4 = 0
2

12x2 – [1 – 6x + 9x2] – 8x – 2 + 6x – 4 = 0
12x2 – 1 + 6x – 9x2 – 8x – 2 + 6x – 4 = 0
3x2 + 4x – 7 = 0
(3x + 7)(x – 1) = 0
x=– x=1

From (1): When x = – , then y = When x = 1, then y =


=4 =–1
6 Solve the simultaneous equations.
Selesaikan persamaan serentak berikut.
x + 5y = 3
+ =6
Give your answers correct to 3 decimal places.
Beri jawapan anda betul kepada 3 tempat perpuluhan.
Ruangan Jawapan / Answer Space [x = 0.410 when y = 0.518 or x = 14.590 when y = – 2.318]
From x + 5y = 3
x = 3 – 5y -------(1)
From + =6

Multiply 2y:  2y +  2y = 6  2y
xy + 6 = 12y
xy – 12y + 6 = 0 ---------(2)
Substitute (1) into (2): (3 – 5y)y – 12y + 6 = 0
3y – 5y2 – 12y + 6 = 0
– 5y2 – 9y + 6 = 0
5y2 + 9y – 6 = 0
a = 5, b = 9, c = – 6
Then, y =  must show this step
= 0.518 or – 2.318
From (1): When y = 0.518, then x = 3 – 5(0.518)
= 0.410
When y = – 2.318, then x = 3 – 5(– 2.318)
= 14.590

83
KERTAS 2 BAHAGIAN C: GERAKAN DALAM GARIS LURUS
PAPER 2 SECTION C: MOTION ALONG A STRAIGHT LINE

Perlu ingat:

Diff Diff

Sesaran, s Halaju, v Pecutan, a


Displacement, s Velocity, v Acceleration, a

Integrate Integrate

Contoh / Example
Diberi halaju, v = 3t 2 – 2, dalam unit ms-1 semasa berada di O, cari sesaran dan pecutan selepas 2 s.
Given the velocity, v = 3t 2 – 2, in unit ms-1 at O, find the displacement and acceleration after 2s.
Penyelesaian / Solution
Cari sesaran, kamirkan halaju. Cari pecutan, bezakan halaju.
Halaju, v = 3t – 2
2
Halaju, v = 3t 2 – 2

Sesaran, s = – 2t + c Pecutan, a = 6t
Apabila t = 2, a = 6(2)
= t3 – 2t + c
= 12 ms-2
Apabila t = 0, s = 0. Maka c = 0
s = t3 – 5t

Apabila t = 2, s = (2)3 – 2(2)


=4m

 Berhenti seketika / stop instantaneously: v = 0


 Pada halaju maksimum / at maximum velocity: a = 0
 Bertukar arah / change its direction: v = 0
 Cari halaju awal atau pecutan awal / find initial velocity or initial acceleration: t = 0
 Pada jarak atau tinggi maksimum / at maximum displacement or height: v = 0
 Bertemu atau berselisih antara satu sama lain / met each other: sesaran sama
 Kembali ke asalan O / return to origin O: s = 0

84
1 A particle P moves in a straight line with velocity, v m s1 is given by v = 4  t2, where t is the time, in seconds, after
passing through O on the line.
Satu zarah, P, bergerak di sepanjang satu garis lurus dengan halaju v ms-1, diberi oleh v = 4  t2, dengan keadaan t ialah masa,
dalam saat, selepas melalui titik tetap O.
Find / Cari,
(a) the initial velocity, in ms-1, of particle P.
halaju awal, dalam ms-1 , zarah P. [1 mark / markah]
(b) the acceleration of P after 4 seconds.
pecutan zarah P selepas 4 saat. [2 marks / markah]
(c) the distance of P from O after 6 seconds.
jarak zarah P dari O selepas 6 saat. [4 marks / markah]
(d) the distance of P from O when P is instantaneous at rest.
jarak P dari O apabila zarah P berhenti seketika. [3 marks / markah]
Ruangan Jawapan / Answer Space [(a) 4, (b) – 8, (c) 48m , (d) m]
(a) At initial velocity, t = 0 s
So, initial velocity = 4  (0)2
= 4 ms-1
(b) From v = 4  t2
a=
= – 2t
So, the acceleration of P after 4 seconds = – 2(4)
= – 8 ms-2
(c) From v = 4  t2
Displacement, s =
s = 4t – +c

When s = 0, t = 0. So, c = 0. Then, s = 4t –

When t = 6, s = 4(6) –
= – 48 m
So, distance P from O is 48 m
(d) When the particle is instantaneous at rest: v=0
4 – t2 = 0
t=2
From s = 4t – , when t = 2, s = 4(2) –
= m

85
2 A stone is projected vertically upwards so that its position above the ground level after t seconds is given by s(t) =
98t  4.9t2 metres, t ≥ 0.
Sebiji batu diluncurkan secara mencancang ke atas dengan keadaan kedudukannya dari lantai selepas t s diberi oleh s(t) = 98t
 4.9t2 meter, t ≥ 0.

Diagram 1 / Rajah 1
(a) Find the distance of the stone from the ground level after 5 s.
Cari jarak batu dari lantai selepas 5 s. [1 mark / markah]
(b) Find the initial velocity of the stone.
Cari halaju awal batu itu. [3 marks / markah]
(c) Find the maximum height reached by the stone.
Cari tinggi maksimum batu itu. [3 marks / markah]
(d) Find the time taken for the stone to hit the ground.
Cari masa yang diambil oleh batu itu untuk menghentam lantai. [2 marks / markah]
(e) What can you say about the acceleration of the stone?
Apa yang boleh anda katakan tentang pecutan batu itu ? [1 mark / markah]
Ruangan Jawapan / Answer Space [(a) 367.5 m, (b) 98 ms , (c) 490 m, (d) 9.8 ms-2]
-1

(a) From s(t) = 98t  4.9t2


When t = 5, s(5) = 98(5) – 4.9(5)2
= 367.5 m
(b) From s(t) = 98t  4.9t2
Velocity, v =
= 98 – 9.8t
At initial velocity, t = 0. Then, v = 98 – 9.8(0)
= 98 ms-1
(c) At maximum height, v = 0. Then, 98 – 9.8t = 0
t = 10 s
So, maximum height, s = 98(10)  4.9(10) 2

= 490 m
(d) Time to hit the ground = 10 s × 2
= 20 s
(e) From v = 98 – 9.8t
Acceleration, a =
= – 9.8
Constant acceleration or deceleration of 9.8 ms-2

86
3 A particle moves along a straight line and passes through a fixed point O. Its velocity, v ms-1, is given by
v = 8 + 2t – t2, where t is the time in seconds after passing through O. The particle stops instantaneously at point M.
Suatu zarah bergerak sepanjang suatu garis lurus dan melalui satu titik tetap O. Halajunya, v ms -1, diberi oleh v = 8 + 2t – t2,
dengan keadaan t ialah masa, dalam saat, selepas melalui O. Zarah itu berhenti seketika di suatu titik M.
[Assume motion to the right is positive / Anggapkan gerakan ke arah kanan sebagai positif.]
(a) the acceleration, in ms-2, of the particle at M.
pecutan, dalam ms-2, bagi zarah itu di M. [3 marks / markah]
-1
(b) the maximum velocity, in ms , of the particle.
halaju maksimum, dalam ms-1, bagi zarah itu. [3 marks / markah]
(c) the total distance, in m, travelled by the particle in the first 10 seconds, after passing through O.
jumlah jarak, dalam m, yang dilalui oleh zarah itu dalam 10 saat pertama, selepas melalui O. [4 marks / markah]
Ruangan Jawapan / Answer Space [(a) – 6 ms-2, (b) 9 ms-1, (c) m]
(a) When stop at M, v = 0
Then, 8 + 2t – t2 = 0
t2 – 2t – 8 = 0
(t – 4)(t + 2) = 0
t=4 t = – 2 (rejected)
Acceleration, a =
= 2 – 2t
When t = 4, a = 2 – 2(4)
= – 6 ms-2
(b) At maximum velocity, a = 0
2 – 2t = 0
t=1
So, the maximum velocity, v = 8 + 2(1) – (1)2
= 9 ms-1

(c) From v = 8 + 2t – t2
Displacement, s = 8t + t2 – +c

When t = 0, s = 0. Then c = 0. So, s = 8t + t2 –

Particle stops at point M when t = 4. Its distance from O, s = 8(4) + (4) 2 –


= m
After t = 4s, the particle changes its direction of motion.
So, total distance travelled in from 4s to 10 seconds, s = [8(10) + (10)2 – ] – [8(4) + (4)2 – ]
= – 180 m

Then, the total distance travelled by the particle in the first 10 seconds = m + 180 m
= m

87
4 A boat travelling in a straight line has its engine turned off at time t = 0. Its velocity in m s-1 after t seconds is then
given by v = , t ms-1, t ≥ 0.
Sebuah bot bergerak di sepanjang garis lurus dan enjinnya dimatikan pada masa t = 0.
Halajunya dalam m s-1 selepas t saat diberi oleh v = , t ms-1, t ≥ 0.
(a) Find the initial velocity of the boat, and its velocity after 3 seconds.
Cari halaju awal dan halaju selepas 3 saat bot itu. [3 marks / markah]
(b) Find how long it takes for the boat to travel 30 metres.
Cari berapa lamakah masa yang diambil oleh bot itu untuk bergerak sejauh 30 m. [5 marks / markah]
(c) Find the acceleration of the boat after 2 seconds.
Cari pecutan bot itu selepas 2 saat. [2 marks / markah]
Ruangan Jawapan / Answer Space [(a) 4 ms-1, (b) 3 s, (c) – 3.125 ms-2]
(a) At initial velocity, t = 0. Then, v =
= 25 ms-1
When t = 3s, v =
= 4 ms-1
(b) From v = , displacement, s =

s= +c

When t = 0, s = 0. So, 0 = +c
c = 50
Displacement, s = + 50

If s = 30, then 30 = + 50

= 20

= (t + 2)
t=3s
(c) From v = ,a=

When t = 2, a =
= – 3.125 ms-2

88
5 Diagram 1 shows the positions and directions of motion of two objects, A and B, moving in a straight line passing
two fixed points, P and Q, respectively. Object A passes the fixed point P and object B passes the fixed point Q
simultaneously. The distance PQ is 90 m.
Rajah 1 menunjukkan kedudukan dan arah gerakan dua objek, A dan B, yang bergerak pada suatu garis lurus dan masing-
masing melalui dua titik tetap P dan Q. Pada ketika A melalui titik tetap P, B melalui titik tetap Q. Jarak PQ ialah 90 m.

Diagram 1 / Rajah 1
The velocity of A, VA ms , is given by VA = 10 + 8t – 2t2, where t is the time, in seconds, after it passes P while B
-1

travels with a constant velocity of  3 m s -1. Object A stops instantaneously at point M.


Halaju A, VA m s-1 , diberi oleh VA = 10 + 8t – 2t2, dengan keadaan t ialah masa, dalam saat, selepas melalui P, manakala B
bergerak dengan halaju malar 3 m s-1. Objek A berhenti seketika di M.
[Assume that the motion towards the right is positive]
[Anggapkan gerakan ke arah kanan sebagai positif]
Find
Cari
(a) the maximum velocity , in, m s-1, of A,
halaju maksimum, dalam ms-1 bagi A. [3 marks / markah]
(b) the distance, in m, of M from P,
jarak, dalam m, M dari P. [4 marks / markah]
(c) the distance, in m, between A and B when A is at the points M.
jarak, dalam m, antara A dan B ketika A berada di titik M. [3 marks / markah]
Ruangan Jawapan / Answer Space [(a) 18 ms-1, (b) m, (c) m]
2
(a) From VA = 10 + 8t – 2t
a = 8 – 4t
At the maximum velocity, a = 0. Then, 8 – 4t = 0
t=2
So, the maximum velocity = 10 + 8(2) – 2(2)2
= 18 ms-1
(b) Particle stops at point M, then vA = 0. So, 10 + 8t – 2t2 = 0
2t2 – 8t – 10 = 0
t2 – 4t – 5 = 0
(t – 5)(t + 1) = 0
t = 5, t = – 1 (rejected)
Displacement, s =
= 10t + 4t2 – +c

When t = 0, s = 0. So, c = 0. Then, s = 10t + 4t2 –

Time to reach P is 5 seconds. So distance M from P = 10(5) + 4(5)2 –


= m

(c) Within 5seconds, distance travelled by object B = 3 × 5


= 15 m
So, the distance between A and B = 90 – – 15
= m

89
6 A particle moves along a straight line and passes through a fixed point O with a velocity of 12 ms-1. The
acceleration, a ms-2, is given by a = 2t – 8, where t is the time, in seconds, after passing through the fixed point O.
Suatu zarah bergerak di sepanjang suatu garis lurus dan melalui satu titik tetap O dengan halaju 12 ms-1. Pecutannya, a ms-2,
diberi oleh a = 2t – 8, dengan keadaan t ialah masa, dalam saat, selepas melalui titik tetap O. [SPM 2013]
[Assume motion to the right is positive]
[Anggapkan gerakan ke arah kanan sebagai positif]
Find
Cari
(a) the initial acceleration, in ms-2, of the particle, [-8]
pecutan awal, dalam ms-2, bagi zarah itu, [1 mark / markah]
-1
(b) the minimum velocity, in ms , of the particle, [-4]
halaju minimum, dalam ms-1, bagi zarah itu, [3 marks / markah]
(c) the time, in seconds, when the particle is instantaneously at rest, [t=2, t=6]
masa, dalam saat, apabila zarah itu berhenti seketika, [2 marks / markah]
(d) the total distance, in m, travelled by the partice in the first 5 seconds. [59/3]
jumlah jarak, dalam m, yang dilalui oleh zarah dalam 5 saat pertama. [4 marks / markah]
Ruangan Jawapan / Answer Space [(a) – 8 ms , (b) – 4 ms , (c) t = 6 t = 2 (d) m]
-2 -1

(a) At the initial acceleration, t = 0


So, the initial acceleration, a = 2t – 8
= 2(0) – 8
= – 8 ms-2
(b) At the minimum velocity, a = 0. Then, 2t – 8 = 0
t=4
Velocity, v =
= t2 – 8t + c
When t = 0, v = 12. So, c = 12. Then, v = t2 – 8t + 12
So, the minimum velocity = (4)2 – 8(4) + 12
= – 4 ms-1
(c) When the particle is instantaneously at rest, v = 0.
Then, t2 – 8t + 12 = 0
(t – 6)(t – 2) = 0
t=6 t=2
(d) From v = t2 – 8t + 12
Displacement, s = – 4t2 + 12t + c

When t = 0, s = 0. Then, 0 = – 4(0)2 + 12(0) + c


c=0
Then, s = – 4t2 + 12t

When t = 5, s = – 4(5)2 + 12(5)


= m

90
7 SPM 2018
Two particles P and Q move along a straight lie such that the displacement of the particles from a fixed point O is s
m. Particle P starts moving from A with sP = 10 + 8t – 8t2 and at the same time the particle Q starts moving from B
with sQ = 6t2 – 9t – 12 where t is the time, in seconds, after the two particles have started moving.
Dua zarah P dan Q bergerak di sepanjang suatu garis lurus dengan keadaan sesaran zarah-zarah itu dari titik tetap O ialah s
m. Zarah P mula bergerak dari A dengan sP = 10 + 8t – 8t2 dan pada masa yng sama zarah Q mula bergerak dari B dengan s Q
= 6t2 – 9t – 12 dengan keadaan t ialah masa, dalam saat, selepas kedua-dua zarah mula bergerak.
(a) Find the distance, in m, between A and B. [2 marks / markah]
Cari jarak, dalam m, antara A dan B.
(b) Find the time, in seconds, when the particles met each other. [3 marks / markah]
Cari masa, dalam saat, apabila kedua-dua zarah itu berselisih.
(c) Calculate the total distance, in m, travelled by particle P when both particles met each other. [5marks]
Hitung jumlah jarak, dalam m, yang dilalui oleh zarah P apabila kedua-dua zarah itu berselisih. [5 markah]
Ruangan Jawapan / Answer Space [(a) 22 m, (b) t = – (rejected), t = 2, (c) 20 m]
(a) When t = 0s
For particle P: sP = 10 + 8(0) – 8(0)2
= 10 m
For particle Q: sQ = 6(0)2 – 9(0) – 12
= – 12 m (To the left)
Distance between A and B = 10 + 12
= 22 m
(b) When the particles met each other from O: sP = sQ
10 + 8t – 8t2 = 6t2 – 9t – 12
0 = 14t2 – 17t – 22
0 = (14t + 11)(t – 2)
t=– (rejected), t = 2

(c) For particle P: sP = 10 + 8t – 8t2


v = 8 – 16t
When v = 0, 0 = 8 – 16(0)
t=½
When t = 2, v = 8 – 16(2)
= – 24
Sketch the graph of v against t:

So, total distance travelled = ½ (8)(0.5) + ½ (1.5)(24)


= 20 m

91
KERTAS 2 BAHAGIAN A/B: JANJANG
PAPER 2 SECTION A/B: PROGRESSION
1 Mr Mahmood wants to buy a car which costs RM 153 300. If he saves his money every months with RM 300 in the
first month, followed by RM 600, RM 1200, …consecutively, find the number of months he could buy the car.
Encik Mahmood ingin membeli sebuah kereta yang berharga RM 153 300. Jika dia menyimpan duit setiap bulan dengan RM
300 pada bulan pertama, diikuti dengan RM 600, RM 1200, … secara berturut-turut, hitung bilangan bulan yang diperlukan
supaya dia dapat membeli kereta tersebut. [4 marks / markah]
Ruangan Jawapan / Answer Space [10]
Form the geometric progression: 300, 600, 1200, ……
So, the first term, a = 300, common ration, r = 2
Then, Tn  153300
n–1
ar  153300
300(2)n – 1  153300
(2)n – 1  511
Taking log10 for both sides: log10(2)n – 1  log10 511
(n – 1) log10 2  log10 511
(n – 1) 
n – 1  8.9972
n  9.9972
n = 10
So, after the 10 months, he could buy the car.

2 Fateeha celebrates her birthday. Her friends brought her a birthday cake as a present as shown in above
Diagram 2 and it is being cut into sectors where the angle of the sector forms an arithmetic progression. Find the
number of cake slices that can be given to her friends.
Fateeha menyambut hari jadinya yang ke-20. Rakannya menghadiahkan sebiji kek yang dipotong sebagaimana ditunjukkan
dalam Rajah 2 dan sudut sektor membentuk satu janjang aritmetik. Cari bilangan potongan kek yang dapat diagihkan kepada
rakan-rakannya. [4 marks / markah]

Diagram 2 / Rajah 2
Ruangan Jawapan / Answer Space [18]
The arithmetic progression, 4º, 4º + d, 4º + 2d, ……., 36º

From Sn = [a + l] where l = las term

360 = [4 + 36]

n = 18

So, the number of slices = 18

92
3 Diagram 3 shows three slices of pizza with the sectors of the circle as a uniform cross-section
Rajah 3 menunjukkan keratan rentas tiga potongan piza yang berbentuk sektor bulatan .

Diagram 3 / Rajah 3
(a) Determine the areas of the sectors above form an arithmetic or a geometric progression. [2marks]
Tentukan sama ada luas sektor-sektor di atas, membentuk janjang aritmetik atau janjang geometri.
(b) If j = 4 cm and the sum of the area of the three sectors are 235.2 cm2, calculate the value of θ. [2 marks]
Jika j = 4 cm dan jumlah luas ketiga-tiga sektor ialah 235.2 cm2, hitung nilai . [2 markah]
Ruangan Jawapan / Answer Space [(a) geometric progression, (b) 4.2]
2
(a) Area of first slide, A1 = ½ (j) (θ) (b) Given the sum of area is 235.2,
= ½ j2θ then ½ j2θ + j2θ + 2j2θ = 235.2
Area of second slide, A2 = ½ (2j)2( ) If j = 4, then ½ (4)2θ + (4)2θ + 2(4)2θ = 235.2
56θ = 235.2
= j2θ
θ = 4.2 rad.
Area of third slide, A3 = ½ (4j)2( )
= 2j2θ

The progression: ½ j2θ, j2θ, 2j2θ, …

Can be deduced the common ratio, r =

=2
So, the areas of the sectors above form a
geometric progression.

4 Jason deposit RM30 in January of each year for 10 years in a bank. The bank gives a compound interest of 5% per
annum at the end of each year.
Jason menyimpan RM30 pada bulan Januari setiap tahun selama 10 tahun dalam sebuah bank. Bank itu memberi faedah
kompaun sebanyak 5% setahun pada setiap hujung tahun.
(a) Calculate the total amount of money he will receive at the end of 10 years.
Hitung jumlah wang yang akan diterimanya pada hujung 10 tahun. [4 marks / markah]
(b) If he decides to stop depositing his money in January in the 5th year, how much money will he receive at the
end of 10 years?
Jika dia berhenti menyimpan wangnya pada bulan Januari pada tahun ke-5, berapakah wang yang akan diterimanya
pada hujung 10 tahun? [3 marks / markah]
Ruangan Jawapan / Answer Space [(a) RM396.20, (b) RM181.94]
(a) By listing method:
End of year 1 End of year 2 End of year 3 End of year 4
30(1.05) = 31.5 (31.5+30)1.05 = 64.575 (64.575+30)(1.05) = 99.3038 (99.3038+30)(1.05) = 135.7690

End of year 5 End of year 6 End of year 7


(135.7690+30)(1.05) = 174.0575 (174.0575+30)(1.05) = 214.2604 (214.2604+30)(1.05) = 256.4734

End of year 8 End of year 9 End of year 10


(256.4734+30)(1.05) = 300.7971 (300.7971+30)(1.05) = 347.3370 (347.3370+30)(1.05) = 396.2039

Total amount of money he will receive at the end of 10 years is RM396.20

93
(b) Starting the fifth year, his money is RM135.7690
By listing method:
End of year 5 End of year 6 End of year 7
(135.7690)(1.05) = 142.5575 (142.5575)(1.05) = 149.6854 (149.6854)(1.05) = 157.1697
End of year 8 End of year 9 End of year 10
(157.1697)(1.05) = 165.0282 (165.0282)(1.05) = 173.2796 (173.2796)(1.05) = 181.9436
If he decides to stop depositing his money in January in the 5 th year, total money he will receive at the end of 10 years
is RM181.94
5 Steven ingin membeli satu komputer Jenama X di sebuah kedai komputer. Harga jualan komputer adalah
RM6546.99 tetapi dia tidak mampu. Pengurus kedai itu menawarkannya untuk membayar secara ansuran bulanan.
Terdapat dua pilihan bayaran ansuran seperti dalam Jadual 6.
Steven wants to buy a computer of Brand X in a computer shop. The selling price of the computer is RM6546.99 and
he cannot afford it. The manager of the shop offered him to pay by monthly installment. There are two options to
choose for the installment as shown in Table 5.
Pilihan 1 Pilihan 2
Option 1 Option 2
Bayaran bulanan secara ansuran selama satu tahun Bayaran bulanan secara ansuran selama dua tahun
dengan bayaran permulaan RM270 dan bertambah dengan bayaran permulaan RM290 dan bertambah
13% setiap bulan daripada bayaran pada bulan RM1 setiap bulan daripada bayaran pada bulan
sebelumnya. sebelumnya.
Monthly installment for one year with starting Monthly installment for two years with starting
payment of RM 270 and an increment of 13% for payment of RM 290 and an increment of RM 1 for
every successive month. every successive month.
Jadual / Table 5
(a) Jika Steven memilih Pilihan 1, hitungkan jumlah yang perlu dibayarnya bagi bayaran bulan terakhir.
If Steven selects Option 1, calculate the amount paid for the last payment.
(b) Hitung jumlah yang perlu dibayar bagi Pilihan 1 dan Pilihan 2.
Find the total amount paid for Option 1 and Option 2.
(c) Jika Steven mempunyai gaji bulanan dan belanja bulanan RM2500 dan RM2000 masing-masing, pilihan mana
yang lebih sesuai buat Steven? Justifikasikan jawapan anda.
If Steven’s monthly salary and expenses are RM2500 and RM2000 respectively, which option is more suitable
to Steven? Justify your answer.
Ruangan Jawapan / Answer Space [(a) 1035.68, (b) Option 1: 6925.55, Option 2: 7236 (c) option 2]
(a) 1st month = RM270 2nd month = 270  = 305.10 3rd month = 305.10  = 344.763 4th month = 344.763  = 389.5822

5th month = 389.5822  = 440.2279 6th month = 440.2279  = 497.4575 7th month = 497.4574  = 562.1268

8th month = 562.1268  = 635.2033 9th month = 635.2033  = 717.7797 10th month = 717.7797  = 811.0911

11th month = 811.0911  = 916.5329 12th month = 916.5329  = 1035.68


Last month payment = RM1035.68
(b) Option 1 Option 2
a = 270 r = 1.13 a = 290, 291, 292, … d = 1
Total payment = Total payment = [2a + (n – 1)d]

= = [2(290) + (24 – 1)(1)]


= 7236
= 6925.55
(c) Highest payment in last month for option 1 is RM1035.68 [Not affordable based on his monthly net salary left RM500
Highest payment in last month for option 2 is = a + (n – 1)d
= 290 + (24 – 1)(1)
= RM313 [Option 2 will be affordable]
TAMAT
END
94

You might also like